Cell Biology

Ace your homework & exams now with Quizwiz!

What are the 2 states in which allosteric enzymes exist?

"R-state" and "T-state"

Oedema can be defined as the accumulation of interstitial fluid in abnormally large amounts. What are the 2 main causes of oedema?

- An increase in hydrostatic pressure - Reduced oncotic pressure (oncotic pressure is osmotic pressure exerted by proteins).

How many molecules of oxygen can myoglobin bind to?

1 (one). Myoglobin can only bind to one oxygen molecule because it has only one heme group. Haemoglobin can bind to 4 oxygen molecules because it has 4 heme groups. Myoglobin has a higher affinity to oxygen at all partial pressures compared with hemoglobin.

How long does it take for insulin to be cleared from the circulation? a. 10 - 15 minutes b. 10 - 15 hours c. 10 - 15 days d. 10 - 15 months

10 - 15 minutes; It's normally cleared from the circulation within ten to fifteen minutes (Guyton 2006). As a result of insulin's rapid action; increases in blood glucose levels can be regulated within a short period of time.

How many molecules of ATP are formed from complete oxidation of 1 palmitic acid molecule? a) 35 b) 104 c) 131 d) 146

131. Complete oxidation of one palmitate molecule (fatty acid containing 16 carbons) generates 129 ATP molecules.

How many carbon atoms does palmitoleic acid have?

16. Palmitoleic acid is an important fatty acid for pharmaceutical applications. It is postulated to have anti-thrombotic effects, which can help prevent stroke (Abraham et al., 1989). At present, it is mainly obtained from Macadamia oil (Macadamia integrifolia), which contains 17% palmitoleic acid.

What's the end product of beta oxidation?

2 acetyl-CoA molecules; Beta oxidation goes on until two acetyl-CoA molecules are produced and the acyl-CoA chain has been completely broken down. In eukaryotic cells, beta oxidation takes place in the mitochondria, whereas in prokaryotic cells, it happens in the cytosol.

The oral glucose tolerance test is used to differentiate between pre-diabetic and diabetic patients. What's the range for normal blood glucose in this test?

7.8 mmol/L and 11.1 mmol/L; At the end of the two hour period a normal blood glucose should be within the range of 7.8 mmol/L and 11.1 mmol/L. Values above 11.1mmol/L suggests impaired glucose tolerance and higher values may be indicative of pancreatic impairment which is common in diseases such as diabetes mellitus.

The oral glucose tolerance is the Gold standard for detecting gestational diabetes. What amount of glucose does the patient ingest for this test?

75g; The oral glucose tolerance test is one of the diagnostic methods for diabetes and reactive hyperglycemia. The test involves the subject ingesting a standard amount of glucose (50g). After ingestion the subject's blood is taken at intervals and the glucose concentration within it quantified

Obesity has been considered as an excess accumulation of body fat, and what is the cut-off value of BMI to consider as obese? a) 25 to 30 b) 30.5 to 34.9 c) >35 d) >40

>35. Obesity is no longer considered a cosmetic issue that is caused by overeating and a lack of self-control. The World Health Organization (W.H.O.), along with National and International medical and scientific societies, now recognize obesity as a chronic progressive disease resulting from multiple environmental and genetic factors.

What technique is used to measure how well your body's cells are able to absorb glucose (sugar) after you consume a specific amount of sugar?

A glucose tolerance test; Doctors use fasting blood sugar levels and hemoglobin A1c values to diagnose type 1 and type 2 diabetes as well as prediabetes. Doctors primarily use a glucose tolerance test to diagnose diabetes during pregnancy (called gestational diabetes).

How many CO2 molecules are exhaled for each O2 molecule utilized in cellular respiration? A. 1 B. 3 C. 6 D. 12

A. 1.

What is the major protein constituent of high-density lipoprotein (HDL)? A. Apolipoprotein A-l B. Apolipoprotein C-l C. Apolipoprotein E D. None of these

A. Apolipoprotein A-l

The active form of glycogen phosphorylase is phosphorylated, while the dephosphorylation of which active form occurs? A. Glycogen synthase B. Glycogen semisynthase C. Glycogen hydrolase D. Glycogen dehydrogenase

A. Glycogen synthase

Why does the glycolytic pathway continue in the direction of glucose catabolism? A. There are essentially three irreversible reactions that act as the driving force for the pathway B. High levels of ATP keep the pathway going in a forward direction C. The enzymes of glycolysis only function in one direction D. Glycolysis occurs in either direction

A. There are essentially three irreversible reactions that act as the driving force for the pathway

The key enzyme in the regulation of fatty acid synthesis is A. acetyl CoA carboxylase B. AMP activated protein kinase C. protein phosphatase D. none of these

A. acetyl CoA carboxylase.

During catabolism, only about 40% of the energy available from oxidizing glucose is used to synthesize ATP. Remaining 60% A. is lost as heat B. is used to reduce NADP C. remains in the products of metabolism D. is stored as fat.

A. is lost as heat.

What happens to acetoacetate during prolonged starvation?

Acetoacetate breaks down spontaneously into acetone, and the acetone is released through the urine and lungs to produce the "acetone breath" that accompanies prolonged fasting. The brain also uses glucose during starvation, but most of the body's glucose is allocated to the skeletal muscles and red blood cells.

Ketone bodies are used by the Brain & heart. What are the 3 ketone bodies released during fat metabolism?

Acetone, Acetoacetic acid & Beta-hydroxybutyric acid

What enzyme is involved in fatty acid synthesis?

Acetyl CoA Carboxylase. Acetyl CoA carboxylase plays an essential role in regulating fatty acid synthesis and degradation. Recall that this enzyme catalyzes the committed step in fatty acid synthesis: the production of malonyl CoA (the activated two-carbon donor).

What is the main molecule that may be produced by the degradation (breakdown) of Carbohydrates, Proteins and Lipids?

Acetyl-CoA. Acetyl-CoA (acetyl coenzyme A) is a molecule that participates in many biochemical reactions in protein, carbohydrate and lipid metabolism. Its main function is to deliver the acetyl group to the citric acid cycle (Krebs cycle) to be oxidized for energy production.

The saturated fatty acyl-CoA molecule will be degraded in a series of four reactions that are part of the β-oxidation. The first reaction involves oxidizing the molecule through the α and β-carbon forming a double bond. The ___________ ______________ enzyme catalyzes this process at the expense of a FAD molecule.

Acyl-CoA dehydrogenase. The product of this process is trans-Δ2-enoyl CoA. In the second step, the trans-Δ2-enoyl CoA will be hydrated in the presence of an enoyl-CoA hydratase forming an L-3-hydroxy acyl CoA. It is then oxidized further to 3-ketoacyl CoA using L-3-hydroxy acyl CoA dehydrogenase as a catalyst. This process proceeds using up one NAD+ to form NADH and H+.

When lipids are not being absorbed, they must be transported from what to maintain metabolism?

Adipose tissues. Lipid transport is a continuously varying process. During the absorption of nutrients from the diet, lipids must be transported to the tissues for use.

What's the carrier of ammonia?

Alanine (Ala). It's the carbon skeleton of pyruvate from muscle to liver. The ammonia is excreted and the pyruvate is used to produce glucose, which is returned to the muscle. Alanine plays a special role in transporting amino groups to liver.

What type of enzymes are characterized by having more than two polypeptide chains, whose subunits have different properties: one isosteric, which is the active site itself, and one allosteric, where enzymatic regulation is performed?

Allosteric enzymes. They are organic chemical substances that are composed with a structure of four molecules, reason why its structure is said to be quaternary.

In ureotelic animals, what's converted into urea in the liver via the urea cycle.

Ammonia (NH3). NH3 is produced from the deamination of amino acids. The urea cycle was the first cycle in biochemistry. It was proposed by Hans Krebs and Kurt Henseleit in 1932.

What drug affects the progression of cytochrome b to cytochrome c in the electron transport chain? a. Amytal b. Orlistat

Amytal. The electron transport chain is series of protein complexes embedded in mitochondrial membrane. It consist of 4 complexes and ATP synthase. The 4 complexes are : 1. NADH dehydrogenase, 2. Succinate dehydrogenase/ FADH, 3. Cytochrome c reductase & 4. Cytochrome oxidase

What is the abnormal buildup of fluid in the abdomen?

Ascites; Technically, it is more than 25 ml of fluid in the peritoneal cavity. Symptoms may include increased abdominal size, increased weight, abdominal discomfort, and shortness of breath. Complications can include spontaneous bacterial peritonitis.

For every one molecule of sugar glucose which is oxidized __________ molecule of pyruvic acid are produced. A. 1 B. 2 C. 3 D. 4

B. 2.

During glycolysis, electrons removed from glucose are passed to A. FAD B. NAD+ C. acetyl CoA D. pyruvic acid

B. NAD+.

What happens after glycolysis when oxygen is available as an electron acceptor? A. Pyruvate is formed B. NADH is produced C. Fermentation D. Oxidative phosphorylation

B. NADH is produced.

In the intestine, the dietary fats are hydrolysed by A. triacylglycerol lipase B. adenylate cyclase C. pancreatic lipase D. protein kinase

B. adenylate cyclase.

HDLs are synthesized in A. blood B. liver C. intestine D. pancreas

B. liver.

A kinase is an enzyme that A. removes phosphate groups of substrates B. uses ATP to add a phosphate group to the substrate C. uses NADH to change the oxidation state of the substrate D. removes water from a double bond

B. uses ATP to add a phosphate group to the substrate.

LDL and HDL are commonly known as __________ and ________ respectively. a. Good cholesterol and bad cholesterol b. Bad cholesterol and good cholesterol c. Assimilatory cholesterol and oxidative cholesterol d. Oxidative cholesterol and assimilative cholesterol

Bad cholesterol and good cholesterol

Arachidonate has 20 carbon atoms with A. 3 double bonds B. 2 double bonds C. 4 double bonds D. 8 double bonds

C. 4 double bonds

The released energy obtained by oxidation of glucose is stored as A. a concentration gradient across a membrane B. ADP C. ATP D. NAD+

C. ATP.

FAD is reduced to FADH2 during A. electron transport phosphorylation B. lactate fermentation C. Krebs cycle D. glycolysis

C. Krebs cycle

In the glycogen synthase reaction, the precursor to glycogen is A. glucose-6-P B. UTP-glucose C. UDP-glucose D. glucose-1-P

C. UDP-glucose. The liver is an important storage site for glycogen.

Glycolysis begins with the phosphorylation of glucose by hexokinase to form glucose-6-phosphate. The enzymes of glycolysis in a eukaryotic cell are located in the A. intermembrane space B. plasma membrane C. cytosol D. mitochondrial matrix

C. cytosol. Fatty acid biosynthesis also takes place in the cytosol. The intermediates are covalently linked to an acyl carrier protein. are synthesized in the cytoplasm from acetyl-CoA.

In most of the naturally occurring monounsaturated fatty acids, the double bonds will be placed between ____________. a. C6 - C7 b. C7 - C8 c. C8 - C9 d. C9 - C10

C9 - C10. In majority of naturally occurring monounsaturated fatty acids, the double bond is between C9 and C10 of the hydrocarbon chain.

Glycolysis, the TCA Cycle, and the Electron Transport/Oxidative Phosphorylation Pathway when functioning together generate a large quantity of ATP by the complete oxidation of glucose to?

CO2 and H2O. There are several enzymatic reactions and/or pathways that utilize carbohydrates that need to be examined. Pathways for: 1. the synthesis of new glucose from three and four carbon metabolic intermediates 2. the synthesis of glycogen from glucose 3. the release of glucose-6-phosphate and glucose from glycogen for entry into metabolism need to be explored.

Organs that have classically been thought to be dependent solely on glucose, such as the brain, can actually use ketones as an alternative energy source. This keeps the brain functioning when glucose is limited. When ketones are produced faster than they can be used, they can be broken down into?

CO2 and acetone. The acetone is removed by exhalation. One symptom of ketogenesis is that the patient's breath smells sweet like alcohol. This effect provides one way of telling if a diabetic is properly controlling the disease. The carbon dioxide produced can acidify the blood, leading to diabetic ketoacidosis, a dangerous condition in diabetics.

What is broken down to glucose to provide energy?

Carbohydrates; It's the third major group of biomolecules. This diverse group is commonly described as sugars, or saccharides, from the Greek word for sugar.

What enzyme limits fatty acid transport into the mitochondrial matrix?

Carnitine Acyltransferase 1. Carnitine palmitoyltransferase 1 (CPT1) is the enzyme in the outer mitochondrial membrane that converts long-chain acyl-CoA species to their corresponding long-chain acyl-carnitines for transport into the mitochondria.

To move the Fatty Acyl-CoA into the mitochondrion, the cytosolic CoA is swapped with a molecule of carnitine. What enzyme catalyzes this reaction?

Carnitine Acyltransferase 1. This reaction occurs on the outer surface of the outer membrane of the mitochondria. On the inner surface of the outer membrane (in the mitochondrial matrix), carnitine is swapped with mitochondrial CoA by the enzyme Carnitine Acyltransferase 2.

What is the precursor for bile salt synthesis? a) Fatty acid b) Glucose c) Cholesterol d) Glycerol

Cholesterol. Bile acid synthesis in mammals and birds occurs in the liver and is the primary pathway in the metabolism of cholesterol, which is the precursor molecule for bile acid synthesis. Bile acids are conjugated in the liver, to glycine or taurine, to inhibit intestinal bile acid resorption and promote lipid metabolism.

What is the substance found in eukaryotic chromosomes that consists of DNA tightly coiled around histones?

Chromatin. Eukaryotic chromosomes contain both DNA and protein tightly packed together to form Chromatin. Chromatin consists of DNA tightly coiled around histones.

Sickle cell anemia is a hereditary disease. It is a blood disorder caused by a point mutation in the allele coding for the beta chain of haemoglobin, particularly a substitution of valine for glutamic acid at position 6 of what chromosome?

Chromosome 11. The result is a defective haemoglobin. It crystallizes readily at low oxygen tension. Hemoglobin is the oxygen-carrying molecule in the red blood cells.

In naturally occurring unsaturated fatty acids, the double bonds are in _______ conformation. a. Cis conformation b. Trans conformation c. A mixture of cis and trans conformation d. Cis and trans conformation alternatively

Cis conformation. This is the conformation of a single bond separating two double bonds in which the double bonds have a dihedral angle of 0 degrees (the double bonds lie in the same plane and point in the same direction).

In the cytosol, what enzyme cleaves citrate to form oxaloacetate and acetyl CoA?

Citrate lyase. It requires ATP and is induced by insulin. The oxaloacetate produced is reduced in the cytosol by NADH, producing NAD+ and malate. The enzyme that catalyzes this reaction is cytosolic malate dehydrogenase.

What are the main intermediates of the Krebs Cycle? (C.I.K.S.S.F.M.O) (Can I Keep Selling Sodas For Money Officer)

Citrate, Isocitrate, Alpha-Ketoglutarate, Succinyl-CoA, Succinate, Fumarate, Malate & Oxaloacetate

What's a common example of a ketohexose?

D-Fructose. In sugar, the D or L designation refers to the configuration of the chiral carbon farthest from the aldehyde or keto group. This is C-5 in glucose. ... The carbon that determines the D and L designations (C-5 in glucose) is also the one that determines the α and β configurations.

What's a common example of an aldopentose?

D-Ribose. Ribose is a kind of sugar that is produced by the body. It is used as a medicine. Ribose is used to improve athletic performance and the ability to exercise by boosting muscle energy. It has also been used to improve symptoms of chronic fatigue syndrome (CFS), fibromyalgia, and coronary artery disease.

What's a common example of an aldohexose?

D-glucose. If the hydroxyl group on the 5th carbon is to the right of the molecule is a D-sugar. If the hydroxyl group on the 5th carbon is to the left of the molecule is L-sugar.

Coenzyme Q is involved in electron transport as A. directly to O2 B. a water-soluble electron donor C. covalently attached cytochrome cofactor D. a lipid-soluble electron carrier

D. a lipid-soluble electron carrier.

Glycolytic pathway regulation involves? A. allosteric stimulation by ADP B. allosteric inhibition by ATP C. feedback, or product, inhibition by ATP D. all of the above

D. all of the above.

A biological redox reaction always involves A. an oxidizing agent B. a gain of electrons C. a reducing agent D. all of these

D. all of these.

The best source of trans fatty acid in diet is ____________. a. Vegetables b. Ground nut c. Dairy products d. Fruits

Dairy products

The solubility of fatty acids in water ________________ a. Increase with increase in chain length and fewer the double bonds b. Increase with increase in chain length and increase in double bonds c. Decrease with chain length and decrease with number of double bonds d. Decrease with chain length and increase with double bonds

Decrease with chain length and decrease with number of double bonds. Fatty acids are partially soluble in water. The partial solubility is due to presence of hydrophilic COOH group at the tip the hydrocarbon chain. The hydrocarbon chains are completely insoluble in water. When the fatty acids are of shorter length, it will be more soluble in the water since the chain length reduction reduces the hydrophobic interactions of hydrocarbons.

What's a common example of a ketotriose?

Dihydroxyacetone. Dihydroxyacetone is a white crystalline powder. In cosmetics and personal care products, Dihydroxyacetone is used in the formulation of self-tanning products. Dihydroxyacetone imparts a colour to the human body. It also enhances the appearance of dry or damaged skin by reducing flaking and restoring suppleness.

Which of the following molecule acts as a sugar carrier in animals? a. Warfarin b. Ubiquinone c. Plastoquinone d. Dolichol

Dolichol. Dolichol is an alcohol functional group containing long chain organic compound made up of many isoprene units. Dolichol has important function in N-glycosylation of proteins in the form of dolichol phosphate. Dolichol also functions as membrane anchor point for the formation of oligosaccharides.

What is the medical term for the buildup of excess fluids in the body's tissues?

Edema; This occurs when the body fails to excrete excess fluid through kidneys or skin.

What is the function of bile salt in the intestine? a) Activator of lipase b) Emulsifier c) Co-factor for cholesteryl esterase d) Inhibitor of lipid absorption

Emulsifier. In the intestine, bile salts function as essential surfactants used to solubilize dietary fats in the hydrophilic milieu of gut. Bile salts are synthesized in the hepatocytes from cholesterol. Bile salts are excreted into the bile and transported, to the intestine, via the intra- and extrahepatic bile ducts.

What term is defined as molecules that are mirror images of each other?

Enantiomers

What term is given to any reaction that requires an input of energy?

Endergonic reaction. Exergonic reactions are also called spontaneous reactions, because they can occur without the addition of energy. Reactions with a positive ∆G (∆G > 0), on the other hand, require an input of energy and are called endergonic reactions.

What type of reaction has a Gibbs Free Energy more than zero?

Endergonic reaction. The reverse process will be favoured. In chemical thermodynamics, an endergonic reaction (also called a heat absorb nonspontaneous reaction or an unfavorable reaction) is a chemical reaction in which the standard change in free energy is positive, and energy is absorbed.

What is the process by which a cell takes the material into the cell by infolding of the cell membrane?

Endocytosis. Endocytosis is the process of taking material into the cell by means of infoldings, or pockets, of the cell membrane. The pocket that results breaks loose from the outer portion of the cell membrane and forms a vacuole within the cytoplasm.

Most commonly occurring fatty acids in nature are: a. Even number of carbon atoms in an unbranched chain of 12 - 24 carbons b. Odd number of carbon atoms in an unbranched chain of 12 - 24 carbons c. Even number of carbon atoms in a branched chain of 12 - 24 carbons d. Odd number of carbon atoms in a branched chain of 12 - 24 carbons

Even number of carbon atoms in an unbranched chain of 12 - 24 carbons. Fatty acids with odd number of carbon, branched chain and more than 24 carbon atoms do occurs in nature but very rarely.

What type of reaction has a Gibbs Free Energy less than zero?

Exergonic reaction. This process is favoured. Exergonic reactions are also called spontaneous reactions, because they can occur without the addition of energy. Reactions with a positive ∆G (∆G > 0), on the other hand, require an input of energy and are called endergonic reactions.

Which of the following is the process which the contents of a cell vacuole are released to the exterior through fusion of the vacuole membrane with the cell membrane? a. Passive diffusion b. Carrier-mediated active transport c. Pinocytosis d. Exocytosis

Exocytosis. Exocytosis is a process by which the contents of a cell vacuole are released to the exterior. This is done so through fusion of the vacuole membrane with the cell membrane.

To which one of the following is the term 'milieu interior' best 'portrayed?' a. Intracellular fluid compartment located within the cell b. Extracellular fluid compartment located outside the cell c. Intracellular fluid compartment located outside the cell d. Extracellular fluid compartment located between the cells

Extracellular fluid compartment located between the cells; Milieu intérieur or interior milieu, from the French, milieu intérieur, is a phrase coined by Claude Bernard to refer to the extra-cellular fluid environment, more particularly the interstitial fluid, and its physiological capacity to ensure protective stability for the tissues and organs of multicellular organisms.

True or False? Ascorbic acid supplements have been very successful in the treatment of Alkaptonuria.

False.

True or False? Late diagnosis and treatment of inborn errors of metabolism can prevent neurological abnormalities.

False. EARLY diagnosis and treatment of inborn errors of metabolism can prevent neurological abnormalities. Many disorders result in neurological abnormalities and mental retardation. Generally, the treatment comprises restricted intake or exclusion of the affected amino acid from the diet.

True or False? Inborn errors of metabolism does not occur when some enzyme involved in metabolism is abnormal.

False. Inborn errors of metabolism occur when some enzyme involved in metabolism is abnormal. The abnormality occurs due to a mutation in the gene encoding the enzyme. The affected enzyme may be absent or deficient.

What enzyme facilitates the conversion of fatty acid to fatty acyl-CoA?

Fatty Acyl-CoA Synthetase. This reaction requires energy in the form of ATP. Fatty Acyl-CoA then becomes fatty acyl-carnitine so that it can transported through the membrane of the mitochondria.

Before Oxidation, Fatty Acids Must be Activated and Transported to the Mitochondrion. What is the fatty acid first converted to?

Fatty Acyl-CoA. This reaction is catalysed by Long Chain Fatty Acyl-CoA Ligase. Fatty Acid Oxidation generates more ATP per carbon than sugars and proceeds 2 carbons at a time. It begins in the cytoplasm.

What is the multi-enzyme protein that catalyzes fatty acid synthesis? a. Fatty acid synthetase b. Pyruvate dehydrogenase c. Acetyl CoA carboxylase d. Protein kinase

Fatty acid synthetase. Fatty acid synthase is a multi-enzyme protein that catalyzes fatty acid synthesis. It is not a single enzyme but a whole enzymatic system composed of two identical 272 kDa multifunctional polypeptides, in which substrates are handed from one functional domain to the next.

What are the building blocks of lipids?

Fatty acids; The building blocks of lipids are: One glycerol molecule and at least one fatty acid, with a maximum of three fatty acids. Glycerol is a sugar alcohol with three OH groups. It acts as a backbone for fatty acids to bond.

What term is given to the metabolic process of converting carbohydrates into acid or alcohol?

Fermentation. It occurs in micro-organisms such as bacteria or fungi. Yeast converts sugar into alcohol to obtain energy. Bacteria converts sugar into lactic acid to obtain energy. Yeast cells are larger than most bacteria.

What term refers to the amount of energy actually available to break and subsequently form other chemical bonds?

Free energy. Bioenergetics or biochemical thermodynamics is the study of the energy changes (transfer and utilizations) accompanying biochemical reactions. Bioenergetics is concerned with the initial and final energy states of the reaction components and not the mechanism of chemical reactions.

What's the hexose sugar found especially in honey and fruit?

Fructose. Fructose, or fruit sugar, is a simple ketonic monosaccharide found in many plants, where it is often bonded to glucose to form the disaccharide sucrose. It is one of the three dietary monosaccharides, along with glucose and galactose, that are absorbed directly into blood during digestion.

Formation of Galactose-1-P from Galactose is catalyzed by a) Hexokinase b) Aldolase c) Galactokinase d) Galactose-1-P Uridyl transferase

Galactokinase. Galactokinase is an enzyme (phosphotransferase) that facilitates the phosphorylation of α-D-galactose to galactose 1-phosphate at the expense of one molecule of ATP.

What's the monosaccharide that's part of the disaccharide lactose?

Galactose. Galactose is a monosaccharide sugar that is about as sweet as glucose, and about 65% as sweet as sucrose. It is a C-4 epimer of glucose. A galactose linked with a glucose comprise lactose.

The electron transport chain consists of a spatially separated series of redox reactions in which electrons are transferred from a donor molecule to an acceptor molecule. What's the underlying force driving these reactions?

Gibbs free energy of reactants and products. The Gibbs free energy is the energy available ("free") to do work. Any reaction that decreases the overall Gibbs free energy of a system is thermodynamically spontaneous.

What term is given to energy that can do work?

Gibbs free energy. Heat is useless as it dissipates. ΔG = Gf (final) - Gi (initial). For a system to be spontaneous, it must either give up energy (decrease in enthalpy), give up order (increase in entropy) or both.

What term is given to the change in free energy?

Gibbs' free energy. Enthalpy and Gibbs Free Energy indicate different things. Enthalpy can tell you about the relative stabilities of the products and reactants. Gibbs free energy however can tell you about whether a reaction is spontaneous (whether a reaction will occur) under a set of specified conditions.

What term is given to the insulin-regulated glucose transporter found primarily in adipose tissues and striated muscle (skeletal and cardiac)?

GluT4 transporter. After a meal, Insulin stimulates the insertion of the GluT4 transporter into the cytoplasmic membrane from their sequestering vesicles. All of the tissues of the body can absorb a large amount of glucose from the blood at a rapid rate and utilize it for energy generation and biosynthesis. Between meals, during a short fast, when blood glucose levels begin to fall, the GluT4 transporter is sequestered again, glucose utilization drops dramatically, and most tissues utilize fatty acids or amino acids to meet their energy needs.

Glycogenesis is the process by which glycogen stored in the liver is broken down to release glucose into the bloodstream. What hormone increases blood glucose levels by stimulating glycogenolysis in the liver?

Glucagon. Glucagon is a peptide hormone, produced by alpha cells of the pancreas. It works to raise the concentration of glucose and fatty acids in the bloodstream and is considered to be the main catabolic hormone of the body. It is also used as a medication to treat a number of health conditions.

The pancreas consists of endocrine and exocrine tissues. The endocrine tissue is grouped together in the islets of Langerhans. What hormone is secreted by alpha cells of the islets of Langerhans?

Glucagon; The hormone functions to increase blood glucose levels by stimulating glycogenolysis in the liver. Glycogenolysis is the process by which glycogen stored in the liver is broken down to release glucose into the bloodstream (Guyton 2006).

What is the metabolic process by which organisms produce sugars for catabolic reactions from non-carbohydrate precursors?

Gluconeogenesis. Glucose is the only energy source used by the brain (with the exception of ketone bodies during times of fasting), testes, erythrocytes, and kidney medulla. In mammals, gluconeogenesis occurs in the liver and kidneys.

In the liver, what enzyme converts glucose 6-phosphate to glucose?

Glucose 6-phosphatase. This is the last step in gluconeogenesis. Glucose is the body's most readily available source of energy. After digestive processes break polysaccharides down into monosaccharides, including glucose, the monosaccharides are transported across the wall of the small intestine and into the circulatory system, which transports them to the liver. In the liver, hepatocytes either pass the glucose on through the circulatory system or store excess glucose as glycogen.

What molecule is produced in the breakdown of glycogen?

Glucose-6-Phosphate. Thus, the breakdown products from glycogen are G1P and glucose (mostly G1P, however). Glucose can, of course, be converted to Glucose-6-Phosphate (G6P) as the first step in glycolysis by either hexokinase or glucokinase. G1P can be converted to G6P by the action of an enzyme called Phosphoglucomutase.

In the pentose phosphate pathway, what enzyme catalyzes the conversion of Glucose-6-phosphate to 6-phosphogluconolactone?

Glucose-6-phosphate dehydrogenase. Water is then added to 6-phosphogluconolactone, giving off a proton (H+), to produce 6-phosphogluconate. Lactonase catalyses this reaction. The pentose phosphate pathway is also called the pentose shunt, the hexose monophosphate shunt or phosphogluconate pathway.

What are the 3 nitrogen carriers of the urea cycle? (G.A.G)

Glutamate, Alanine & Glutamine

Which of the nitrogen carriers of the urea cycle transfers one amino group WITHIN cells? a. Glutamate b. Glutamine c. Alanine

Glutamate. Alanine plays a special role in transporting amino groups to liver. Ala is the carrier of ammonia and of the carbon skeleton of pyruvate from muscle to liver. The ammonia is excreted and the pyruvate is used to produce glucose, which is returned to the muscle.

What's the immediate precursor of both ammonia (through oxidative deamination by glutamate dehydrogenase) and aspartate nitrogen, through transamination of oxaloacetate by aspartate aminotransferase (AST).

Glutamate. The carbon and oxygen of urea are derived from CO2. Urea is produced by the liver, and then is transported in the blood to the kidneys for excretion in the urine.

Which of the nitrogen carriers of the urea cycle transfers 2 amino groups between cells? a. Glutamate b. Glutamine c. Alanine

Glutamine. It then releases its amino group in the liver. Alanine transfers amino group from tissue (muscle) into the liver. Alanine is a neutral amino acid.

The most common aldoses include the 6-carbon sugars glucose, mannose, and galactose. What's a common example of an aldotriose?

Glyceraldehyde. Glyceraldehyde is a triose monosaccharide with chemical formula C3H6O3. It is the simplest of all common aldoses. It is a sweet, colourless crystalline solid that is an intermediate compound in carbohydrate metabolism.

What's the most basic amino acid?

Glycine. As a neurotransmitter, glycine both stimulates and inhibits cells in the brain and central nervous system, affecting cognition, mood, appetite and digestion, immune function, pain perception, and sleep. Glycine is also involved in the production of other biochemicals that influence these body functions.

What term is given to the process of converting glucose-6-phosphate to glycogen?

Glycogenesis.

What term is given to the process of converting glycogen to glucose-6-phosphate?

Glycogenolysis.

What are the membrane carbohydrates that are covalently bonded to lipids; derived from sphingosine; which contains 1 or more sugar residues?

Glycolipids. Glycolipids are lipids with a carbohydrate attached by a glycosidic (covalent) bond.

What term is given to the process of converting glucose-6-phosphate to pyruvic acid?

Glycolysis. Glycolysis literally means "splitting sugars" and is the process of releasing energy within sugars. In the presence of oxygen, glycolysis is the first stage of cellular respiration. Glycolysis takes place in the cytosol of the cell's cytoplasm.

The electrons that enter the transport chain are from NADH and FADH2 molecules produced during earlier stages of cellular respiration. In cellular respiration, what process occurs before the Citric Acid Cycle?

Glycolysis. The Krebs cycle uses the two molecules of pyruvic acid formed in glycolysis and yields high-energy molecules of NADH and flavin adenine dinucleotide (FADH2), as well as some ATP. Glycolysis occurs in the cytosol while the Krebs cycle occurs in the mitochondrion.

What is the end product of beta oxidation in the peroxisomes?

H2O2. Beta oxidation in the peroxisomes yields H2O2 instead of FADH2 and NADH, producing heat as a result.

Where does most of the fatty acid synthesis occur?

Hepatocytes. Adipose tissue consists of adipocytes (fat cells). Adipocytes are metabolically very active, responding to hormonal stimuli in a metabolic interplay with the liver, skeletal muscles, and the heart. Adipose tissue convert glucose into fatty acids, from which TAGs are made and stored.

Individuals that are _________ for sickle cell anaemia tend to have an increased resistance to malaria caused by Plasmodium palcifarum.

Heterozygous.

What term is given to substances in the biological system which on hydrolysis yields free energy equal to or greater than that of ATP i.e. ∆G= -7.3 kcal/mol?

High energy/ energy rich compounds. The high energy compounds have anhydride bonds. Compounds which liberate less than 7.3 kcal/mol (lower than ATP hydrolysis to ADP + Pi) are referred to as low energy compounds. •ATP is the principal carrier of chemical energy in the cell and consists of an adenine, a ribose and a triphosphate.

Triacylglycerols are transported in body fluids by molecules called lipoproteins. What's the blood fat that helps to transport cholesterol out of the arteries, thereby protecting against heart disease?

High-density lipoprotein (HDL). High-density lipoprotein (HDL) cholesterol is known as the "good" cholesterol because it helps remove other forms of cholesterol from your bloodstream. Higher levels of HDL cholesterol are associated with a lower risk of heart disease. It's carried through your bloodstream attached to proteins.

What is the normal intermediate in the degradation of phenylalanine and tyrosine that is associated with Alkaptonuria?

Homogentisate. Alkaptonuria is transmitted as a single recessive Mendalian trait. Homogentisate accumulates in alkaptonuria because homogentisate oxidase is absent and thus its degradation is blocked. After accumulating, Homogentisate is then excreted in the urine causing it to be dark.

What is the enzyme responsible for the breakdown of triglycerides into fatty acids and monoacylglycerol in the intestine? a) Pancreatic lipase b) Lipoprotein lipase c) Hormone-sensitive lipase d) Phospholipase

Hormone-sensitive lipase. Hormone-sensitive lipase (HSL, EC 3.1. 1.3) is an amphiphilic enzyme and the key control of energy substrate flow in mammals. Its activity in adipose tissue determines the rate of hydrolysis of stored triacylglycerols and thereby the production of fatty acids for release as free fatty acids (FFAs) into the circulation.

Lipids are water insoluble, organic greasy (oily) substances due to the presence of their long hydrocarbon tails. What type of lipid can be broken down into smaller sections to get energy?

Hydrolyzable lipids. Hydrolyzable lipids are those that contain a functional group that will react with water. The functional group is usually an ester and the list of compounds includes neutral fats, waxes, phospholipids, and glycolipids.

What term describes a solution in which the solute concentration is high outside of the cell?

Hypertonic. This causes water to move out of the cell resulting in the cell shrinking. In isotonic solutions, the concentrations of solute inside and outside the cell are equal

Where does the hexose monophosphate shunt occur?

In the cytosol. In order to obtain the Glucose-6-phosphate required for the pentose shunt, glycogen is converted to glucose 1-phosphate by glycogen phosphorylase. Glucose 1-phosphate is then converted to glucose 6-phosphate by phosphoglucomutase. The pentose shun then produces ribose and NADPH.

What stimulates the liver to store glucose in the form of glycogen?

Insulin, A large fraction of glucose absorbed from the small intestine is immediately taken up by hepatocytes, which convert it into the storage polymer glycogen. Insulin has several effects in the liver which stimulate glycogen synthesis.

What pancreatic hormone is released by the beta cells of the islets of Langerhans and is associated with energy abundance?

Insulin; It is secreted in an effort to store the excess energy that is accumulating due to the intake of carbohydrates and sugars. Insulin stimulates rapid uptake, storage and increased use of glucose by the tissues especially the muscles, adipose tissue and liver.

The general configuration of a lipoprotein involves having the hydrophobic region inside the core of the sphere, while the superficial layer is composed of the hydrophilic regions. What type of lipoprotein is formed from the degradation of very-low-density lipoproteins?

Intermediate Density Lipoprotein (IDL). It enables fats and cholesterol to move within the bloodstream. In general, IDL, somewhat similar to low-density lipoprotein (LDL), transports a variety of triglyceride fats and cholesterol and, like LDL, can also promote the growth of atheroma. VLDL is a large, triglyceride-rich lipoprotein secreted by the liver that transports triglyceride to adipose tissue and muscle.

What term refers to the all the energy within a given system, including the kinetic energy of molecules and the energy stored in all of the chemical bonds between molecules?

Internal energy. Internal energy is defined as the energy associated with the random, disordered motion of molecules. It is separated in scale from the macroscopic ordered energy associated with moving objects; it refers to the invisible microscopic energy on the atomic and molecular scale.

What accounts for 2/3 of the Total Body Water?

Intracellular fluid. The remaining 1/3 is extracellular fluid Interstitial. By weight, the average human adult male is approximately 60% water, and the average adult female is approximately 55% water. There can be considerable variation in body water percentage based on a number of factors like age, health, water intake, weight, and sex.

During prolonged starvation, what does the brain metabolize?

Ketone bodies. During starvation, most tissues utilise fatty acids and/or ketone bodies to spare glucose for the brain. Glucose utilisation by the brain is decreased during prolonged starvation as the brain utilises ketone bodies as the major fuel. High concentrations of ketone bodies result in significant excretion of ketones.

Glucose is oxidized to provide almost all of the ATP used by the brain. What does the brain resort to as an emergency backup fuel when carbohydrates are limited?

Ketone body metabolism. Energy sources in the brain vary with nutritional states. The brain cannot directly use free fatty acids or lipids from the blood as fuels, it will use ketone body from fatty acids. ATP is required to create and maintain an electrical potential across the plasma membrane of neurons

Polysaccharides serve as energy storage (e.g., starch and glycogen) and as structural components (e.g., chitin in insects and cellulose in plants). What are the 3 carbon precursors for gluconeogenesis?

Lactate, pyruvate and glycerol. Glucose is released from stored glycogen and is synthesized by the liver to meet the needs of the glucose-dependent tissues. GLUCONEOGENESIS is the synthesis of "new" glucose from three or four carbon precursors. The three-carbon precursors for gluconeogenesis are lactate, pyruvate, and glycerol.

When glucose levels are plentiful, the excess acetyl CoA generated by glycolysis can be converted into fatty acids, triglycerides, cholesterol, steroids, and bile salts. What's the name of this process?

Lipogenesis. It creates lipids (fat) from the acetyl CoA and takes place in the cytoplasm of adipocytes (fat cells) and hepatocytes (liver cells). When you eat more glucose or carbohydrates than your body needs, your system uses acetyl CoA to turn the excess into fat. Although there are several metabolic sources of acetyl CoA, it is most commonly derived from glycolysis.

To obtain energy from fat, triglycerides must first be broken down by hydrolysis into their two principal components, fatty acids and glycerol. What's the name of this process?

Lipolysis. It takes place in the cytosol. The resulting fatty acids are oxidized by β-oxidation into acetyl CoA, which is used by the Krebs cycle. The glycerol that is released from triglycerides after lipolysis directly enters the glycolysis pathway as DHAP. Because one triglyceride molecule yields three fatty acid molecules with as much as 16 or more carbons in each one, fat molecules yield more energy than carbohydrates and are an important source of energy for the human body.

Urea synthesis in mammals takes place primarily in tissues of the:

Liver. Urea is synthesized in liver and is released into the blood and cleared by kidneys in the urine. Urea synthesis in the liver involves five enzymes: (1) Carbamoyl phosphate synthetase 2) Ornithine carbamoyltransferase (3) Argininosuccinate synthetase (4) Argininosuccinate lyase and (5) Arginase.

What is the cell organelle filled with enzymes that is needed to break down certain materials in the cell?

Lysosomes. Lysosomes are organelles that carry the cell's digestive enzymes. Lysosomes have to be membrane bound. This is because of the enzymes. The digestive enzymes in the cell break down and "recycle" the unneeded materials in the cell.

Sugars that differ only by the configuration around 1 carbon atom are known as epimers of one another. What is the C2 epimer of glucose?

Mannose Mannose, packaged as the nutritional supplement "d-mannose", is a sugar monomer of the aldohexose series of carbohydrates. It is a C-2 epimer of glucose. Mannose is important in human metabolism, especially in the glycosylation of certain proteins.

Carbon atoms in fatty acid are __________ than those of sugars. a. Less reduced b. More reduced c. Less oxidized d. More oxidized

More reduced. Carbon atoms in fatty acids are more reduced than those in sugars. This is why fatty acids are more energy rich than sugars.

The lipid digestion process is regulated by different local hormones. The cholecystokinin hormone released from a) Mucosa of jejunum b) Pancreatic delta cells c) Gastric Parietal cells d) Pancreatic alpha cells

Mucosa of jejunum. Secretin acts in tandem with another hormone called cholecystokinin (CCK). Not only does CCK stimulate the pancreas to produce the requisite pancreatic juices, it also stimulates the gallbladder to release bile into the duodenum.

What type of lipid cannot be broken down?

Non-Hydrolyzable lipids.

Which 3 amino acids, when added to the buffered medium, greatly accelerates the rate of urea formation?

Ornithine, Citrulline & Arginine. Amino acids have an amino group an a carboxyl group.

Glutamate is metabolically converted to a alpha-ketoglutarate and NH4+ by what process?

Oxidative deamination. Oxidative deamination is a form of deamination that generates α-keto acids and other oxidized products from amine-containing compounds, and occurs only in the liver.

The heart is normally aerobic and obtains its energy from which process?

Oxidative phosphorylation. Failure of O2 to reach a portion of the heart muscle when the blood vessels are blocked by lipid deposits (atherosclerosis) or blood clots (coronary thrombosis) can cause this region of the heart muscle to die, a process known as myocardial infarction.

What enzyme, produced by the pancreas, releases free cholesterol from cholesteryl esters in the small intestine?

Pancreatic Cholesteryl Ester Hydrolase. Once the micelles of free fatty acids, 2-monoacylglycerols, and bile acids become small enough, they can be absorbed from the intestinal lumen into the body. Inside the body the fatty acids are esterified to re-form triacylglycerols. These triacylglycerols combine with lipoproteins released by the intestines to produce chylomicrons, which act as serum transport particles for triacylglycerols.

The anti-obesity drug Orlistat inhibits a) Pancreatic lipase b) Lipoprotein lipase c) Hormone-sensitive lipase d) Phospholipase

Pancreatic lipase. Orlistat is a drug used in the treatment of obesity. Its primary function is preventing the absorption of fats from the human diet, thereby reducing caloric intake. Orlistat works by inhibiting pancreatic lipase, an enzyme that breaks down triglycerides in the intestine. Without this enzyme, triglycerides from the diet are prevented from being hydrolyzed into absorbable free fatty acids and are excreted undigested.

What term is given to the process by which an ion or molecule passes through a cell wall via a concentration gradient, or from an area of high concentration to an area of low concentration?

Passive diffusion

What is the most common inborn error disease of amino acid metabolism?

Phenylketonuria (PKU). It has an incidence of about 1 in 10, 000 live births. It was the first inborn error of amino acid metabolism to be treated successfully by diet manipulation. In PKU, there is a block in the conversion of phenylalanine into tyrosine. When the level of phenylalanine exceeds 1 mmol/L, alternate metabolites of phenylalanine are formed.

What enzyme catalyzes the the conversion of oxaloacetate to PEP (phosphoenolpyruvate)?

Phosphoenolpyruvate kinase. Phosphoenolpyruvate carboxykinase (PEPCK) is an enzyme in the lyase family used in the metabolic pathway of gluconeogenesis. It converts oxaloacetate into phosphoenolpyruvate and carbon dioxide. It is found in two forms, cytosolic and mitochondrial.

what is the structural arrangement of the phospholipid bilayer?

Plasma membrane layers composed of phospholipid molecules arranged with polar heads facing the outside and nonpolar tails facing the inside.

Which of the following product of Triglyceride breakdown and subsequent beta-oxidation would undergo gluconeogenesis? a) Propionyl Co A b) Acetyl CoA c) Acetoacetate d) Beta-hydroxybutyrate

Propionyl Co A. Propionyl-CoA is later converted into succinyl-CoA through propionyl-CoA carboxylase (PCC) by the use of vitamin B12. Propionyl-CoA is not only produced from the oxidation of odd-chain fatty acids, but also by the oxidation of amino acids including methionine, valine, isoleucine, and threonine

What type of oedema is characterized by fluid accumulation in the lungs; caused by Hydrostatic Pressure; raised pressure across pulmonary capillaries; respiratory fatigue and failure?

Pulmonary Oedema; Pulmonary oedema is fluid accumulation in the tissue and air spaces of the lungs. It leads to impaired gas exchange and may cause respiratory failure. ... Pulmonary oedema, especially acute, can lead to fatal respiratory distress or cardiac arrest due to hypoxia. It is a cardinal feature of congestive heart failure.

What molecule serves as the cofactor for transamination reactions?

Pyridoxal phosphate (PLP). PLP is derived from vitamin B6. Glutamate and alpha-ketoglutarate are often involved in transamination reactions, serving as one of the amino acid/alpha-ketoacid pairs. Transamination reactions are readily reversible and can be used in the synthesis or the degradation of amino acids.

Which of the following enzymes is Thiamine dependent and essential for glucose oxidation in the brain? a) Pyruvate dehydrogenase complex b) Acetyl co A carboxylase c) Transaldolase d) Succinyl-co A Thiokinase

Pyruvate dehydrogenase complex. Brain glucose metabolism depends on three thiamine-dependent enzymes: transketolase (TK), the pyruvate dehydrogenase complex (PDHC) and the α-ketoglutarate dehydrogenase complex (KGDHC).

Fats and oils are the most common forms of lipids. Fats are abundantly found in?

Reproductive tissue; Fats are found in many places. One well-known form of fat is found in human and animal tissue. The primary function of fats is energy storage.

Each amino acid contains a central carbon, a hydrogen, a carboxyl group, an amino group, and a variable R group. What is the endomembrane system covered with ribosomes where many proteins for transport are assembled?

Rough Endoplasmic Reticulum. The endomembrane system is composed of the different membranes that are suspended in the cytoplasm within a eukaryotic cell. These membranes divide the cell into functional and structural compartments, or organelles. In eukaryotes the organelles of the endomembrane system include: the nuclear membrane, the endoplasmic reticulum, the Golgi apparatus, lysosomes, vesicles, endosomes, and plasma membrane.

Which of the following class of fatty acids can be directly absorbed from intestine? a) Very long chain fatty acid b) Long-chain fatty acid c) Short-chain fatty acid d) Cholesterol esters

Short-chain fatty acid. About 95 percent of lipids are absorbed in the small intestine. ... Despite being hydrophobic, the small size of short-chain fatty acids enables them to be absorbed by enterocytes via simple diffusion, and then take the same path as monosaccharides and amino acids into the blood capillary of a villus.

At room temperature (25oC), a fat with saturated fatty acid of 12 - 20 carbon have: a. Liquid consistency b. Solid consistency c. Cannot be predicted

Solid consistency. Saturated fatty acids do not have double bonds and hence molecules can be packed compactly. When unsaturation is present in the fatty acids, due to double bonds there will be kinks in the fatty acid chains and this hinders the compact packing of fatty acid residues. Since the molecules are not compactly packed in the unsaturated fats, they acquire a liquid consistency.

What are the 3 fatty acids that are most commonly present in the triglycerides of the human body? (St.O.P)

Stearic acid Oleic acid & Palmitic acid. These fatty acid chains may range from 14 - 24 carbons atoms in length but 16 & 18 carbons are most common.

What is the type of arterial embolism that originates commonly from thrombi in the diseased heart, especially in the left ventricle?

Systemic Embolism. These diseases of heart include myocardial infraction, cardiomyopathy, RHD, congenital heart disease, infective endocarditis, and prosthetic cardiac valves. These arterial emboli invariably cause infarction at the sites of lodgement which include, in descending order of frequency, lower extremity, brain, and internal visceral organs (spleen, kidneys, intestines). Thus, the effects and sites of arterial emboli are in striking contrast to venous emboli which are often lodged in the lungs.

What are the most abundant bile salts?

Taurocholate and glycocholate

What are the 3 main examples of bile acids (See Tiddies) (C.T.D)?

Taurocholate, Cholate & Deoxycholate. The stomach has a small surface area to volume ratio, and therefore many of the triacylglycerols are not accessible to the enzymes. The small intestine has mechanisms for emulsifying lipids. The process begins by dispersing the lipid aggregates mechanically as a result of the muscles of the small intestine forcing the partially digested material through the relatively small spaces of the intestinal lumen.

Which law states that "energy can be converted from one form to another with the interaction of heat, work and internal energy, but it cannot be created nor destroyed, under any circumstances"?

The First Law of Thermodynamics. This is the law of conservation of energy. It was put forward by Robert Mayer in 1941. With the interactions of heat, work and internal energy, there are energy transfers and conversions every time a change is made upon a system.

What is the term given to the pressure of water to move, typically into the capillary, as the result of the presence of plasma proteins?

The Oncotic pressure. Some large proteins in blood plasma can move into and out of the endothelial cells packaged within vesicles by endocytosis and exocytosis. Water moves by osmosis.

Which law states that "a system and its surroundings always proceed to a state of maximum disorder or maximum entropy"?

The Second Law of Thermodynamics. It's also called law of the degradation of energy or law of entropy. This law was developed in 1850s by German Physicist Rudolf Clausius. Entropy (S) is the tendency of systems of molecules to randomization.

What process generates the majority of ATP in most organisms?

The electron transport chain. The function of the electron transport chain is to produce a transmembrane proton electrochemical gradient as a result of the redox reactions. If protons flow back through the membrane, they enable mechanical work, such as rotating bacterial flagella. ATP synthase, an enzyme highly conserved among all domains of life, converts this mechanical work into chemical energy by producing ATP, which powers most cellular reactions.

What is the Pasteur Effect?

The increase in the rate of glycolysis which occurs under partially anaerobic conditions. Under anaerobic conditions, the rate of glucose metabolism is faster, but the amount of ATP produced is smaller.

What type of reaction has a Gibbs Free Energy equal to zero?

The reaction is at equilibrium. ΔG = ΔH - T ΔS; ΔG - Gibbs free energy change (KJ/mol), ΔH - Enthalpy change (KJ/mol), T - Temperature (Kelvin) & ΔS - Entropy change (KJ/mol)

Fatty acids are stored in adipocytes as triglycerides and can be degraded in the same location, if needed. In which organ of the body does the digestion triacylglycerols begin?

The stomach. The majority of lipids in a normal diet are present in the form of triacylglycerols. Digestion of these compounds begins in the stomach, which contains acid-stable lipases that release some free fatty acids from dietary triacylglycerols. However, the stomach is not capable of efficiently cleaving triacylglycerols, because these hydrophobic molecules tend to aggregate, and the lipases are only capable of hydrolyzing the triacylglycerols at the surface of the aggregates.

What's the purpose for the oral glucose tolerance test?

The test challenges the activity of the pancreas in regulating blood glucose, mainly through the secretion of insulin. After the glucose concentrations are measured a glucose tolerance curve is plotted with glucose concentration against time. At 2 hours after consuming 75g of glucose, normal blood glucose level is between 7.8 mmol/L and 11.1 mmol/L or below 140mg/dl after.

What does an abnormal oral glucose tolerance test suggest?

There may be an underlying pathology which may be related to pancreatic function. Therefore the test provides a starting point which can then be explored for further investigations. The subject of our test appears to have a normal carbohydrate metabolism. After fasting the glucose ingested was metabolized normally and within the allotted time period.

What enzyme catalyzes the final step of beta oxidation?

Thiolase. A cysteine residue in the enzyme attacks the β-keto carbon atom. Bond cleavage occurs producing the enolate of acetyl CoA and a thioester intermediate. The final product of β-oxidation is a 2-carbon shorter acyl-CoA. These β-oxidation steps are successively repeated until enough acetyl CoA units are produced.

What accounts for 60% of body weight in adults?

Total body water. By weight, the average human adult male is approximately 60% water, and the average adult female is approximately 55% water. There can be considerable variation in body water percentage based on a number of factors like age, health, water intake, weight, and sex.

In which class of reactions is pyridoxal phosphate a cofactor?

Transamination reactions. Pyridoxal phosphate acts as a coenzyme in all transamination reactions, and in some oxylation and deamination reactions of amino acids. The aldehyde group of pyridoxal phosphate forms a Schiff-base linkage with the epsilon-amino group of a specific lysine group of the aminotransferase enzyme.

The plasma membrane mediates cellular processes by regulating the materials that enter and exit the cell. What name is given to integral proteins that do not extend all the way through the membrane?

Transmembrane proteins. A different integral protein may not extend all the way through the plasma membrane. Instead, these integral proteins may need to be bound to a membrane so that their product is easy to expel. Some of the proteins responsible for producing neurotransmitters operate in this way. This allows the product to be amassed where it is needed most, at the very tips of the neurons where the signal can be released.

What are the 2 major forms of stored energy in vertebrates?

Triacylglycerols (TGs) and glycogen. Glycogen can supply ATP for muscle contraction for less than an hour. Sustained work is fueled by metabolism of TGs which are very efficient energy stores because: (1) they are stored in an anhydrous form & (2) their fatty acids are more reduced than amino acids or monosaccharides.

Most animal tissues contain appreciable amounts of lipid. When in the form of depot fat it consists largely of: a) Cholesterol esters b) Phosphatides c) Chylomicra d) Triglycerides

Triglycerides ✓

__________________ yield more than twice the energy per unit mass when compared to carbohydrates and proteins.

Triglycerides. Therefore, when glucose levels are low, triglycerides can be converted into acetyl CoA molecules and used to generate ATP through aerobic respiration. The breakdown of fatty acids, called fatty acid oxidation or beta (β)-oxidation, begins in the cytoplasm, where fatty acids are converted into fatty acyl CoA molecules. This fatty acyl CoA combines with carnitine to create a fatty acyl carnitine molecule, which helps to transport the fatty acid across the mitochondrial membrane.

True or False? Deficiency or genetic mutation of the N-Acetylglutamate synthase enzyme (autosomal recessive) will cause urea cycle failure.

True. This is a severe neonatal disorder with fatal consequences, if not detected immediately upon birth. Hyperammonemia and general hyperaminoacidemia in a newborn are conditions in which the liver contain no detectable ability to synthesize N-acetylglutamate. Early symptoms include lethargy, vomiting, and deep coma

True or False? In alkaptonuria, urine turns dark on standing as the homogentisate is oxidized and polymerized to a melanin like substance.

True. This leads to pigmentation of connective tissue. Pigmented spots may be seen on sclera and ears. The treatment of alkaptonuria is symptomatic.

True or False? In born errors of metabolism may occur in metabolism of all nutrients including amino acids.

True. When an enzyme is absent or deficient, metabolism of the concerned amino acid becomes abnormal. Over 50 inborn errors of metabolism of amino acids have been discovered. The clinical abnormalities may occur due to: Decreased synthesis of products, Accumulation of intermediates & Formation of alternate metabolites

True or False? Some relatively common inborn errors of metabolism are: Alkaptonuria, Phenylketonuria (PKU) & Maple Syrup Urine Disease.

True. Alkaptonuria was the first inborn error of metabolism to be identified. It is an inborn error of tyrosine metabolism. It's an inherited metabolic disorder caused by the absence of homogentisate oxidase.

What are the main risk factors of obesity?

Type 2 diabetes mellitus, Increased blood pressure, Coronary Heart Disease & Stroke

What's the most common form of diabetes?

Type 2 diabetes. A failure to remove excess glucose from the bloodstream results in increased amounts of glucose which exerts a large amount of osmotic pressure in the extracellular fluid which can lead to dehydration. Long-term increases in blood glucose may cause damage to tissues, especially blood vessels

Adipocytes store TAGs arriving from the liver as?

Very Low Density Lipoproteins. They also store TAGs arriving from the intestinal tract. In the fasting state, when fuel is needed, TAGs stored in adipose tissue are hydrolyzed by lipases to release free fatty acids, and delivered to skeletal muscles and the heart. Release of FA from adipocytes is accelerated by glucagon or epinephrine, which stimulates triacylglycerol lipase.

What does Saponification of a mixture of vegetable fats with alcoholic KOH yield?

Water soluble derivatives of glycerol; The products of a saponification are a soap and glycerol. Soaps are water soluble sodium or potassium salts of fatty acids.

Cells in the body take up the circulating glucose in response to insulin and, through a series of reactions called glycolysis, transfer some of the energy in glucose to ADP to form ATP. What is the proportion of ATP produced by oxidative phosphorylation? a) 60% b) 70% c) 80% d) 90%

a) 60%. Glycolysis produces only 2 ATP molecules, but somewhere between 30 and 36 ATPs are produced by the oxidative phosphorylation of the 10 NADH and 2 succinate molecules made by converting one molecule of glucose to carbon dioxide and water, while each cycle of beta oxidation of a fatty acid yields about 14 ATPs.

A fatty acid not stored in human fat depots is: a) Arachidonic b) Oleic c) Palmitic d) Stearic

a) Arachidonic. Fatty acids are synthesized in the cytosol. It's very similar to the reverse of Beta oxidation. The difference between them is that in the synthesis, fatty acids are carried by ACP; instead of the CoA group in beta oxidation.

Bile acid is derived from: a) Cholesterol b) Amino acids c) Fatty acids d) Bilirubin

a) Cholesterol. Bile acids make up a group of sterol-derived compounds that act as detergents in the intestine to facilitate the digestion and absorption of fats and fat-soluble molecules. In mammalian species, the cholesterol side chain is trimmed to yield C24-sterol derivatives. In other vertebrate species, the hydroxylation of the side chain does not lead to its removal and the products of the biosynthetic pathway are referred to as bile alcohols.

The magnitude of proton motive force depends on a) Energy charge of cell b) Permeability of membrane c) Number of ribosomes in cell d) None

a) Energy charge of cell. The proton motive force occurs when the cell membrane becomes energized due to electron transport reactions by the electron carriers embedded in it. Basically, this causes the cell to act like a tiny battery. Its energy can either be used right away to do work, like power flagella, or be stored for later in ATP.

Which one of the following statements regarding body fluid composition is correct? a) Extracellular fluid contains mainly Sodium ions & Chloride ions b) Extracellular fluid contains mainly Potassium and Chloride Ions c) Intracellular fluid contains mainly Sodium ions and Bicarbonate ions d) Intracellular ions contains mainly Potassium and Calcium ions

a) Extracellular fluid contains mainly Sodium ions & Chloride ions

Which of the following statements is false about gluconeogenesis? a) From the hydrolysis of tri-acyl-glycerol, fatty acids can be used as a carbon source b) From red blood cells, lactate can be used as a carbon source c) From the hydrolysis of tri-acyl-glycerol, glycerol is converted to glucose in gluconeogenesis d) From muscle vigorous muscle activity, lactate can be used as a carbon source

a) From the hydrolysis of tri-acyl-glycerol, fatty acids can be used as a carbon source. Fatty acids and ketogenic amino acids cannot be used to synthesize glucose. The transition reaction is a one-way reaction, meaning that acetyl-CoA cannot be converted back to pyruvate. As a result, fatty acids can't be used to synthesize glucose, because beta-oxidation produces acetyl-CoA.

The effect of increased levels of hydrogen ions in the intermembrane space of the mitochondria is a) Increase ATP production b) Decreased levels of oxidative phosphorylation c) Increased levels of water in intermembrane space d) Decreased levels of chemiosmosis

a) Increase ATP production. The high concentration of hydrogen ions in the intermembrane space is used to power ATP synthase as these hydrogen ions are allowed to flow back through a channel in ATP synthase.

Which oil is not produced in the body a) Linoleic b) oleic c) palmitic d) Stearic

a) Linoleic.

Which of the following is essential fatty acid? a) Linolenic acid b) Arachidonic acid c) Oleic acid d) Palmitic acid

a) Linolenic acid. The term essential fatty acid refers to a fatty acid which the body cannot manufacture, and must obtain from dietary sources. These essential fatty acids were originally designated as Vitamin F, until it was realized that they must be classified with the fats.3 There are two fatty acids designated as essential fatty acids: linoleic acid and alpha-linolenic acid.

Lipases are generally classified as intracellular or extracellular. Which of the following are extracellular lipases? a) Pancreatic lipase & Lipoprotein lipase b) Hormone-sensitive lipase (in adipocytes) & Lysosomal lipases c) Phospholipase

a) Pancreatic lipase & Lipoprotein lipase. The general configuration of a lipoprotein involves having the hydrophobic region inside the core of the sphere, while the superficial layer is composed of the hydrophilic regions. At different parts of the digestive tract, metabolism of triacylglycerols may occur. The mechanism of metabolism is facilitated by a group of enzymes called lipases.

Lipid metabolism begins in the intestine where ingested triglycerides are broken down into smaller chain fatty acids and subsequently into monoglyceride molecules by a) Pancreatic lipase b) Lipoprotein lipase c) Hormone-sensitive lipase d) Phospholipase

a) Pancreatic lipase. When food reaches the small intestine in the form of chyme, a digestive hormone called cholecystokinin (CCK) is released by intestinal cells in the intestinal mucosa. CCK stimulates the release of pancreatic lipase from the pancreas and stimulates the contraction of the gallbladder to release stored bile salts into the intestine. CCK also travels to the brain, where it can act as a hunger suppressant.

Which of the following lipids act as lungs surfactant? a) Phosphatidylcholine b) Phosphatidylethanolamine c) Ceramide d) Phosphatidylinositol

a) Phosphatidylcholine. The lung surfactants contain a complex mixture of phospholipids, neutral lipids, and specific proteins. The lung surfactants are essential for normal lung function as it reduces surface tension at the air-liquid interface of alveolar spaces. Phosphatidylcholine (or dipalmitoylphosphatidylcholine) is the major component of the surfactant comprising 90% of the lipids.

In the pentose phosphate pathway, the major products are a) Ribulose and NADPH b) Ribulose and NADH c) Ribulose and NAD+ d) Ribulose and ATP

a) Ribulose and NADPH. The products formed are a pentose, ribulose and NADPH. The pentose phosphate pathway takes place in the cytosol of the cell, the same location as glycolysis. The two most important products from this process are the ribose-5-phosphate sugar used to make DNA and RNA, and the NADPH molecules which help with building other molecules.

What concentration of glucose in the blood does the body strive to maintain? a. 1mg/mL. b. 10mg/mL. c. 100mg/mL. d. 1000mg/mL.

a. 1mg/mL. Four tissues are dependent upon glucose alone for energy generation. Red blood cells are absolutely glucose-dependent since they have only glycolysis for energy generation. In the fed state, nervous tissue, adrenal medulla, and testis/ovaries use only glucose for their energy generation. In the starvation state, these three tissues can adapt to other energy sources if the starvation comes on slowly and is prolonged.

D-Glyceraldehyde is an example of? a. 3-carbon D-Aldoses b. 4-carbon D-Aldoses c. 5-carbon D-Aldoses d. 6-carbon D-Aldoses

a. 3-carbon D-Aldoses. The 4 carbon D-aldoses are D-Erythrose and D-Threose. The 5 carbon D-aldoses are: D-Ribose, D-Arabinose, D-Xylose and D-Lyxose.

Dihydroxyacetone is a a. 3-carbon D-Ketose b. 4-carbon D-Ketose c. 5-carbon D-Ketose d. 6-carbon D-Ketose

a. 3-carbon D-Ketose.

All of the following would be likely to increase in diabetes except a. 3-hydroxy-3-methyl-glutaryl-CoA b. Acetoacetate c. D-Beta -hydroxybutyrate d. Acetyl CoA

a. 3-hydroxy-3-methyl-glutaryl-CoA.

What's the water content of the Brain? a. 80-85% b. 75-80% c. 70-75% d. 8-10%

a. 80-85%. Water content varies in different body organs and tissues, from as little as 8 percent in the teeth to as much as 85 percent in the brain.

In the regulation of acetyl-CoA carboxylase in the synthesis of fatty acids, which enzyme is responsible for converting the active carboxylase to an inactive form? a. AMP-activated protein kinase b. Caspase

a. AMP-activated protein kinase. It does so by phosphorylating the active carboxylase.

What is the common fate of these amino acids: Leucine, Lysine, Phenylalanine, Tryptophan & Tyrosine? a. Acetoacetyl CoA b. Acetyl-CoA c. Pyruvate

a. Acetoacetyl CoA.

Before fatty acids can pass the mitochondrial membrane in Beta oxidation, they are activated with coenzyme A (CoA), thus, forming Acyl-CoA. Which enzyme catalyzes this reaction? a. Acyl-CoA synthetase b. Carnitine acyltransferase 1 c. Translocase d. Carnitine acyltransferase 2

a. Acyl-CoA synthetase. This reaction consumes ATP.

Which of the following glucogenic amino acids form Pyruvate? a. Alanine, Glycine, Threonine, Cysteine & Serine b. Arginine, Proline, Histidine & Glutamine c. Asparagine & Aspartate d. Tyrosine & Phenylamine

a. Alanine, Glycine, Threonine, Cysteine & Serine. The pyruvate produced can either be incorporated in the process of glycolysis to form Acetyl-CoA or be utilized by Pyruvate carboxylase to form Oxaloacetate in gluconeogenesis.

What is characteristic of the primary structure of proteins? a. Amino acid sequence b. Beta-sheets and alpha-helix c. 3-D folding d. Interaction of enzyme with protein

a. Amino acid sequence The simplest level of protein structure, primary structure, is simply the sequence of amino acids in a polypeptide chain. Each chain has its own set of amino acids, assembled in a particular order.

Which of the following is an autosomal recessive metabolic disorder that results in the inability to condense citrulline with aspartate? a. Argininosuccinate synthase deficiency b. Ornithine transcarbamylase deficiency c. Carbamoyl Phosphate Synthetase deficiency

a. Argininosuccinate synthase deficiency (aka. Citullinemia). This condition causes and accumulation of blood and excretion in the urine. Type 1 citrullinemia usually becomes evident in the first few days of life. In Type 2 citrullinemia, the signs and symptoms usually appear during adulthood and mainly affect the nervous system.

Each of the following statements about bile salts is correct EXCEPT which one? a. Bile salts are amphipaths, but do not form micelles in the intestine b. Bile salts are made in the liver from cholesterol c. Bile salts help disperse dietary lipid in the intestine and thereby aid in digestion d. Taurocholic acid and glycocholic acid are major bile salts

a. Bile salts are amphipathic, but do not form micelles in the intestine. Bile salts help to form micelles of triglycerides in the intestines to boost their digestion by water-soluble lipases. They can act as acids (bile acids) or as salts when conjugated with amino acids. Hence their amphipathic nature.

A dipeptide, which consists of beta-alanine and histidine and is found in muscle is: a. Carnosine b. Creatine c. Bradykinin d. None of these

a. Carnosine. Carnosine is found in highest concentrations in the brain and muscle tissue. As a zwitterion, it acts as a buffer, aiding the tissues in defense against oxidative damage.Fun fact: Carnosine has been shown to increase the Hayflick limit in fibroblasts--- which gives us a clue into creating immortal cell lines. It has been shown to act as a geroprotector-- protecting against aging and senescence.

Gallstones usually consist mostly of: a. Cholesterol b. Bile pigments c. Bile salts d. Mucin

a. Cholesterol. Most gallstones are made of cholesterol that precipitates out of solution. This happens when there is too much cholesterol than the phospholipids and bile salts can manage to keep in solution as they're moving from the liver into bile.

In precursor biosynthesis, which is a major area of carbon metabolism in which the Krebs Cycle is involved, which of the precursors is responsible for the production of fatty acids & sterols? a. Citrate b. Alpha-Ketoglutarate c. Succinyl-CoA d. Malate

a. Citrate.

In the electron transport chain, NADH dehydrogenase complex (NADH to Ubiquinone) is a. Complex 1 b. Complex 2 c. Complex 3 d. Complex 4

a. Complex 1. The flow of electrons through Complex I to ubiquinone to Complex III is accompanied by the movement of protons from the mitochondrial matrix to the outer (cytosolic) side of the inner mitochondrial membrane (the intermembrane space).

If one wished to diagnose myocardial infarction, one would best measure isoenzymes of: a. Creatinine kinase b. Lactate dehydrogenase c. Alkaline phosphatase d. Alcohol dehydrogenase

a. Creatinine kinase (CK). CK is present normally in the brain, heart and lungs. Each of its three isozymes is specific to that location and is more useful than LDH for diagnosing a heart attack.In particular, levels of CK-MB are elevated during a myocardial infarction.Of course in recent times we have found an even more specific test-- namely the blood tests for troponin and myoglobin.

Which of the following is a chemical species with an affinity for a negative charge? a. Electrophile b. Nucleophile

a. Electrophile

Hexokinase is inhibited by which of the following? a. Glucose 6 Phosphate b. Fructose c. Insulin d. Glucagon

a. Glucose 6 Phosphate. Hexokinase, the enzyme catalyzing the first step of glycolysis, is inhibited by its product, glucose 6-phosphate. However, the liver, in keeping with its role as monitor of blood-glucose levels, possesses a specialized isozyme of hexokinase called glucokinase that is not inhibited by glucose 6-phosphate.

What's the product of glycogen breakdown? a. Glucose-6-phosphate b. Ribose-5-phosphate c. Pyruvate

a. Glucose-6-phosphate.

Which of the following amino acids is the precursor for GABA & Glutathione? a. Glutamate b. Arginine c. Glycine d. Phenylalanine

a. Glutamate.

Eukaryotic cells are larger than prokaryotic cells and have a "true" nucleus, membrane-bound organelles, and rod-shaped chromosomes. In which of the following does the sorting of lipids and proteins takes place? a. Golgi apparatus b. Nucleus c. Smooth Endoplasmic Reticulum d. Rough endoplasmic reticulum

a. Golgi apparatus. The nucleus houses the cell's DNA and directs the synthesis of proteins and ribosomes. Mitochondria are responsible for ATP production; the endoplasmic reticulum modifies proteins and synthesizes lipids; and the golgi apparatus is where the sorting of lipids and proteins takes place. Peroxisomes carry out oxidation reactions that break down fatty acids and amino acids and detoxify poisons; vesicles and vacuoles function in storage and transport.

What type of oedema leads to swelling which can cut off blood supply? a. Interstitial oedema b. Intracellular oedema

a. Interstitial oedema; The occlusion of blood supply can result in intracellular oedema. Pulmonary interstitial oedema represents a form of pulmonary oedema resulting from pathological fluid buildup in the interstitial spaces due to increased hydrostatic driving pressure.

Which of the following enzymes occurs as both NAD+- and NADP+- linked forms? a) Isocitrate dehydrogenase b) Succinate dehydrogenase c) Carbonic anhydrase d) Aconitase

a. Isocitrate dehydrogenase; In eukaryotes, isocitrate dehydrogenase exists in two forms: an NAD+-linked enzyme found only in mitochondria and displaying allosteric properties, and a non-allosteric, NADP+-linked enzyme that is found in both mitochondria and cytoplasm . The enzyme from some species can also use NAD+ but much more slowly

All of the following statements about 1,25-dihydroxycholecalciferol are correct EXCEPT which one? a. It is synthesized from the 25-hydroxycholecalciferol in liver b. It is synthesized from 25-hydroxycholecalciferol in kidney c. It stimulates the intestinal absorption of dietary calcium d. Its synthesis is stimulated by parathyroid hormone.

a. It is synthesized from the 25-hydroxycholecalciferol in liver. 1,25-dihydroxycholecalciferol also known as 1,25-dihydroxy Vitamin D, is synthesized in the kidneys, from its friend 25-hydroxy vitamin D. It is released when calcium levels in the serum are low. Parathyroid hormone is secreted in response to low calcium levels. It causes the synthesis of 1,25-dihydroxy cholecalciferol from 25-hydroxy cholecalciferol in the kidneys which boosts the absorption of calcium from the diet.

The reduction of pyruvate to form L-lactate is catalyzed by what enzyme? a. Lactate dehydrogenase b. Alcohol dehydrogenase c. Pyruvate dehydrogenase d. Pyruvate decarboxylase

a. Lactate dehydrogenase. Lactate dehydrogenase (LDH or LD) is an enzyme found in nearly all living cells. LDH catalyzes the conversion of lactate to pyruvate and back, as it converts NAD+ to NADH and back. A dehydrogenase is an enzyme that transfers a hydride from one molecule to another.

What precursor of gluconeogenesis is obtained from the constant anaerobic glycolysis in the Red Blood Cell and the occasional anaerobic glycolysis in Skeletal Muscle? a. Lactate b. Pyruvate c. Glycerol

a. Lactate. Gluconeogenesis is the pathway by which glucose is formed from non-hexose precursors such as glycerol, lactate, pyruvate, and glucogenic amino acids. Lactate generated from anaerobic glycolysis gets shunted to the liver. Glucose enters the cell through specific carriers and must be immediately phosphorylated, as dephosphorylated glucose is poisonous to the cell.

Which of the following amino acids are exclusively ketogenic? a. Leucine & Lysine b. Lysine, proline, and threonine c. Asparagine & Aspartate d. Tyrosine & Phenylamine

a. Leucine & Lysine. Amino acids that can form intermediates of carbohydrate metabolism can subsequently be converted to glucose via gluconeogenesis. These amino acids are called glucogenic amino acids. Amino acids that are converted to acetoacetyl-CoA or acetyl-CoA, which can be used for the synthesis of ketone bodies but not glucose, are called ketogenic amino acids.

Which enzyme is used to convert fructose 6-phosphate to fructose 1,6-bisphosphate in the committed 3rd step of glycolysis? a. Phosphofructokinase-1 b. 3-phosphoglycerate kinase c. Hexokinase d. Phosphoglucose isomerase

a. Phosphofructokinase-1 (PFK-1). PFK-1 is able to regulate glycolysis through allosteric inhibition, and in this way, the cell can increase or decrease the rate of glycolysis in response to the cell's energy requirements. For example, a high ratio of ATP to ADP will inhibit PFK-1 and thus glycolysis. (Also, note the difference between bisphosphate and biphosphate; bisphosphate refers to 2 different carbons and biphosphate would be 2 substituents on the same carbon atom.

Which of the following groups of compounds includes only lipids of appreciable importance in the structure of cell membranes? a. Phospholipids, sphingolipids, cholesterol b. Triglycerides, free fatty acids, cholesterol c. Glycolipids, prostaglandins, cholesterol esters d. Triglycerides, phospholipids, cholesterol esters

a. Phospholipids, sphingolipids, cholesterol.

Fatty acids are the building blocks of lipids. What term is given to the group of bioactive, hormone-like chemicals derived from fatty acids? a. Prostaglandins b. Right Ventricular Failure c. Cardiomegaly d. Embolism

a. Prostaglandins. They have a wide variety of biological effects including roles in inflammation, platelet aggregation, vascular smooth muscle dilation and constriction, cell growth, protection of from acid in the stomach, and many more. Fatty acids are unbranched carbon chains that make up most lipids.

The enzyme which catalyzes the conversion of pyruvate to oxaloacetate a) Pyruvate carboxylase b) Pyruvate dehydrogenase c) Pyruvate kinase d) Phosphofructokinase-1

a. Pyruvate carboxylase. Pyruvate carboxylase (PC) is a mitochondrial enzyme bound to biotin that catalyzes the conversion of pyruvate to oxaloacetate when abundant acetyl CoA is available, replenishing Krebs cycle intermediates in the mitochondrial matrix. PC is involved in gluconeogenesis, lipogenesis and neurotransmitter synthesis. Pyruvate carboxylase requires the coenzyme biotin to convert pyruvate to oxaloacetate.

For any enzymic reaction, when the ratio [ES]/[E] increases: a. The velocity of the reaction will increase b. The velocity of the reaction will decrease c. The Km is increased d. The turnover number decreases

a. The velocity of the reaction will increase. We can work out that an increased [ES]/[E] ratio is indicative of a lower Km. Then we can also work out that Km is the substrate concentration required to reach half the maximum velocity. A lower Km--> less substrate is needed to push the reaction to the max. Thus the reaction would proceed a little faster (Note that Vmax would not be changed).

Which of the following is true regarding hormones? a. They are the first messengers of a multi-component signal b. None of the answers are true c. They work in the region of the body where they are made d. All of the answers are true e. They all cross the lipid bilayer by themselves

a. They are the first messengers of a multi-component signal.

Which of the following is true regarding hormone receptors? a. They change shape on binding to their hormone b. They change shape on binding to another receptor c. None of the answers are true d. All of the answers are true e. They are always membrane bound

a. They change shape on binding to their hormone

Which of the following is an excess fluid that is protein poor due to increased capillary pressure? a. Transudate b. Exudate

a. Transudate. It will have a low specific gravity <1.012, a fluid protein to serum ratio <0.5, and LDH 1.012, a fluid protein to serum ratio of >0.5, and LDH >0.6. It is thus likely to arise from conditions that lead to low levels of proteins, such as malnutrition and protein-losing enteropathies.

Which of the following amino acids is the precursor for Niacin and Melatonin? a. Tryptophan b. Histidine c. Glycine d. Phenylalanine

a. Tryptophan

One of the important metabolic processes that occur in fatty acids is β-oxidation. Here, short fatty acid chains are transported into the mitochondrial matrix as free acids. As they enter the mitochondrial matrix, they are activated by the enzyme ___________ ____________ to convert it into acyl-CoA. a. Acyl-CoA synthetase b. Carnitine acyltransferase 1 c. Translocase d. Carnitine acyltransferase 2

a. acyl-CoA synthetase. Long chains of fatty acids move to inner parts of the mitochondrial membrane and are converted to acyl-carnitine. The acyl group of the molecule is then transferred to CoA to form acyl-CoA.

In beta oxidation, Acyl-CoA is oxidized to enoyl-CoA under the conversion of FAD (flavin adenine dinucleotide) to FADH2. This reaction is catalyzed by a. acyl-coa dehydrogenase b. enoyl-coa hydratase c. hydroxyacyl-CoA dehydrogenase d. ketothiolase

a. acyl-coa dehydrogenase.

The proteins of the Major Histocompatibility Complex (MHC) bind and display: a) antigen fragments b) B cell fragments. c) immunoglobulin fragments. d) macrophage fragments.

a. antigen fragments. Major histocompatibility complex (MHC) class I molecules bind peptides derived from cellular proteins and display them for surveillance by the immune system. These peptide-binding molecules are composed of a heavy chain, containing an antigen-binding groove, which is tightly associated with a light chain (beta 2-microglobulin).

In the next step of the first phase of glycolysis, which enzyme converts glucose-6-phosphate into fructose-6-phosphate. a. glucose-6-phosphate isomerase b. Phosphofructokinase-1 c. Glyceraldehyde-3-phosphate dehydrogenase

a. glucose-6-phosphate isomerase. Like glucose, fructose is also a six carbon-containing sugar.

In the urea cycle, what enzyme converts glutamate in the cell to alpha-ketoglutarate and ammonia? a. glutamate dehydrogenase b. carbamoyl phosphate synthetase c. ornithine transcarbamylase d. argininosuccinate synthetase

a. glutamate dehydrogenase. The glutamate dehydrogenase of mammalian liver has the unusual capacity to use either NAD+ or NADP+ as cofactor. The NH+4 produced forms carbamoyl phosphate. The reaction requires 2 ATP and bicarbonate ion. Most dehydrogenases are specific for NAD+ as electron acceptor, some, such as glucose-6-phosphate dehydrogenase, require NADP+. Glutamate dehydrogenase, can react with either NAD+ or NADP+.

The arrangement of sugars into D- and L- configurations is based upon their resemblance to D- and L-: a. glyceraldehyde b. tartaric acid c. glucose d. ribose

a. glyceraldehyde.

In the binding of oxygen to myoglobin, the relationship between the concentration of oxygen and the fraction of binding sites occupied can best be described as: a) hyperbolic. b) linear with a negative slope. c) linear with a positive slope. d) sigmoidal.

a. hyperbolic. Myoglobin exists either in an oxygen free-form called deoxymyoglobin or in an oxygen bound form called oxymyoglobin. Whether myoglobin binds to oxygen depends on the presence of the prosthetic group, heme. When myoglobin is able to bind to oxygen, it serves as the primary oxygen-carrying molecule in muscle tissue

What term is defined as the loss of large amounts of plasma protein, usually albumin, through urine due to increased permeability of the glomerular membrane which can result in oedema? a. Nephrotic Syndrome b. Anaemia c. Pentose Phosphate Pathway d. Lean Body Mass

a. nephrotic syndrome. Nephrotic syndrome is a kidney disorder that causes your body to excrete too much protein in your urine. Nephrotic syndrome is usually caused by damage to the clusters of small blood vessels in your kidneys that filter waste and excess water from your blood.

Allosteric enzyme regulation occurs at rate-determining steps primarily. For example, following meal glycolysis in the liver is stimulated by the increase of fructose-2, 6-bisphosphate, which is an allosteric activator of what enzyme? a. Phosphofructokinase-1 b. 3-phosphoglycerate kinase c. Hexokinase d. Phosphoglucose isomerase

a. phosphofructokinase-1 (PFK-1). In addition, the opposing pathway of gluconeogenesis is turned off by fructose-2, 6-bisphophate allosterically inhibiting fructose-1,6-bisphosphatase.

The formation of Vitamin D3 (cholecalciferol) from 7 dehydrocholesterol requires a: a. photochemical step b. hydroxylation in kidney c. hydroxylation in liver d. hydroxylation in both kidney and liver

a. photochemical step. 7-dehydrocholesterol is converted to cholecalciferol b UV light (e.g in the sun). In the kidneys we have inactive vitamin D (25-hydroxycholecalfierol) converted to calcitriol (1,25-dihydroxycholecalciferol) or active vitamin D.

What term is given to the property of cell membranes that allows some substances to pass through while others cannot? a. Selectively Permeable b. Heteroplasmy c. Apoptosis d. Cytoskeleton

a. selectively permeable. Permeability describes the ease with which substances can pass through a border, such as a cell membrane. Permeable means that most substances can easily pass through the membrane. Impermeable means that substances cannot pass through the membrane.

Glucagon binds to high-affinity G-protein-coupled receptors that reside on the membranes of hepatocytes, resulting in the activation of the cyclic AMP (cAMP)-producing what enzyme? a. Phosphofructokinase-1 b. 3-phosphoglycerate kinase c. adenylyl cyclase d. Phosphoglucose isomerase

adenylyl cyclase. This second messenger acts to activate cAMP-dependent protein kinase A (PKA), which results in the phosphorylation-mediated activation or inhibition of the regulatory enzymes involved in carbohydrate and lipid metabolism. Note that the receptors that bind glucagon are different than the types that bind epinephrine or insulin and are not found in skeletal muscle.

The urea cycle, also known as the ornithine cycle, is a cycle of biochemical reactions that produces urea from ammonia. In the urea cycle one nitrogen of the urea molecule is supplied by free NH3, and the other nitrogen by?

aspartate. Urea is the major disposal form of amino groups derived from amino acids, and accounts for about 90% of the nitrogen-containing components of urine. Urea is produced by the liver mainly in the cytosol but with the initial steps occuring in the mitochondria.

Total amount of body fluid in extracellular compartment in 70 kg. Male subject is: a) 38% b) 22% c) 17% d) 5%

b) 22%

Which enzyme cleaves the 6-carbon Fructose 1,6-bisphosphate (FBP) into 2 different 3-carbon molecules, GAP, and dihydroxyacetone phosphate (DHAP). a) Hexokinase b) Aldolase c) Galactokinase d) Galactose-1-P Uridyl transferase

b) Aldolase. This reaction is an aldol cleavage (retro aldol condensation)that features an enolate intermediate stabilized by resonance. Aldol cleavage of G6P would produce products of unequal carbon chain length, whereas, aldol cleavage of FBP results in 2 interconvertible 3-carbon fragments that can enter a common catabolic pathway.

NADP is a cofactor used in a) Catabolic reactions b) Anabolic reactions c) Elimination reaction d) Redox reactions

b) Anabolic reactions. Anabolic reactions are those that require energy to occur. Cells can couple anabolic reactions together with catabolic ones to form an efficient energy cycle; the catabolic reactions transform chemical fuels into cellular energy, which is then used to initiate the energy-requiring anabolic reactions.

The urea molecule consists of two nitrogens and one carbon atom. The first nitrogen atom is derived from the free ammonium ion and second from an amino acid. The amino acid that donates the second amino group for the formation of urea is a) Arginine b) Aspartate c) Glutamate d) Ornithine

b) Aspartate.

Which of the following amino acid: keto acid pair bridge the urea cycle with the tricarboxylic acid cycle for the maintenance of nitrogen and amino acid pool a) Glutamate: Alpha-ketoglutarate b) Aspartate: Oxaloacetate c) Alanine: Pyruvate d) None of the above

b) Aspartate: Oxaloacetate

Pyruvate carboxylase requires ————— as a coenzyme. a) FMN b) Biotin c) NAD+ d) TPP

b) Biotin

Which of the following only uses glucose as its source of energy? a) Red Blood Cells b) Brain c) Heart d) Liver

b) Brain. Glucose is virtually the sole fuel for the human brain, except during prolonged starvation. The brain lacks fuel stores and hence requires a continuous supply of glucose.

Lipases are generally classified as intracellular or extracellular. Which of the following are intracellular lipases? a) Pancreatic lipase & Lipoprotein lipase b) Hormone-sensitive lipase (in adipocytes) & Lysosomal lipases c) Phospholipase

b) Hormone-sensitive lipase (in adipocytes) & Lysosomal lipases. Hormones control the activity of hormone-sensitive lipase. An example of this is the increase in lipase activity when an individual suffers from stress. The stress triggers a release of the hormones adrenaline/noradrenaline.

Insulin enhances the uptake of triacylglycerols in adipose tissues. Which of the following enzyme facilitates the uptake? a) Hormone-sensitive lipase b) Lipoprotein lipase c) LCAT d) Apo C-II

b) Lipoprotein lipase. Insulin biosynthesis involves the production of two inactive precursors preproinsulin and proinsulin, which will be subsequently cleaved to form the active hormone upon a rise in blood glucose levels. Insulin causes glucose uptake in both myocytes and adipocytes and directs these cells to synthesize glycogen, protein, and triacylglycerols. These actions are mediated by insulin's binding to the α-subunit of the insulin receptor, which produces a cascade of cell-signaling β-subunit of insulin receptor substrate (IRS) proteins.

What enzyme plays a critical role in breaking down fat in the form of triglycerides, which are carried from various organs to the blood? a) Pancreatic lipase b) Lipoprotein lipase c) Hormone-sensitive lipase d) Phospholipase

b) Lipoprotein lipase. When lipoprotein lipase breaks down triglycerides, the fat molecules are used by the body as energy or stored in fatty tissue for later use.

The deficiency of which of the following enzymes urea cycle results in impaired the synthesis of urea, accumulation of ammonia (hyperammonemia) and orotic aciduria? a) Carbamoyl Phosphate synthase I b) Ornithine Transcarbamoylase c) Argininosuccinate synthase d) Arginosuccinase

b) Ornithine Transcarbamoylase. Ammonia toxicity results in the conversion of alpha-ketoglutarate to glutamate by the enzyme glutamate dehydrogenase. Glutamate is then converted to glutamine by the enzyme glutamine synthetase.

Which of the following compound is administered to the patients with urea cycle defect to remove the ammonia? a) Inulin b) Phenylbutyrate c) Arginine d) Ornithine

b) Phenylbutyrate.

Which one of the following statements regarding body composition in a young adult male is correct? a) Water is 75% of the body weight b) Protein is about 18% of body weight c) Fat is about 5% of body weight d) Electrolytes are 2% of the body weight

b) Protein is about 18% of body weight. Often in medicine, a mineral dissociated from a salt that carries an electrical charge (an ion) is called and electrolyte. For instance, sodium ions (Na+) and chloride ions (Cl-) are often referred to as electrolytes.

The rate of electron transfer when proton motive force is high is a) Faster b) Slower c) Moderate d) None

b) Slower. The proton-motive force created by the pumping out of protons by the respiratory chain complexes is in the mitochondria of most tissues mainly used to translocate protons through the ATP synthase complex, leading to the formation of ATP from adenosine diphosphate (ADP) and phosphate.

The first step in the urea cycle is a condensation of CO2, ammonia, and ATP to form carbamoyl phosphate. The following are true regarding the formation of carbamoyl phosphate except: a) The reaction is catalyzed by a rate-limiting enzyme carbamoyl phosphate synthase-I b) The presence N-acetyl glutamate inhibits the activity of CPS I enzyme c) Two molecules of ATP are required for this reaction d) The reaction occurs in mitochondria

b) The presence N-acetyl glutamate inhibits the activity of CPS I enzyme. Ammonia toxicity leads to the depletion of alpha-ketoglutarate from the central nervous system. This inhibits the krebs cycle and the production of ATP. Neurotransmitters, such as glutamate (excitatory neurotrasmitter) and GABA (inhibitory neurotransmitter, may contribute to CNS effects.

What is the main product of cholesterol metabolism? a) vitamin d b) bile salts c) fatty acids d) carbon dioxide

b) bile salts. Bile acids are derived from cholesterol. Cholic acid and some other bile acids are synthesized from cholesterol in the liver. Taurocholate is produced through conjugation of cholate with taurine; similarly, glycocholate is produced through conjugation with glycine.

In glycolysis, the glyceraldehyde-3-phosphate is further phosphorylated with groups donated by dihydrogen phosphate present in the cell to form what three-carbon molecule? a. pyruvate b. 1,3-bisphosphoglycerate c. PEP d. glucokinase

b. 1,3-bisphosphoglycerate. The energy of this reaction comes from the oxidation of (removal of electrons from) glyceraldehyde-3-phosphate. In a series of reactions leading to pyruvate, the two phosphate groups are then transferred to two ADPs to form two ATPs. Thus, glycolysis uses two ATPs but generates four ATPs, yielding a net gain of two ATPs and two molecules of pyruvate.

What percentage of the total oxygen consumed by a resting human adult is used by the brain? a. 0% b. 20% c. 50% d. 100%

b. 20%. The brain uses energy for transmission of impulses. It normally uses only glucose as fuel. Since it has little glycogen, it depends on incoming glucose from the blood. If the blood glucose falls below a critical level for even a short period of time, severe and son=sometimes irreversible changes in the brain function may occur.

Amylopectin is similar to glycogen in that they both have alpha-1,4-glycosidic linkages and alpha-1,6-glycosidic branches. Amylopectin, however, has branches ever ____________________, while glycogen has branches every_________________. a. 8-12 units, 24-30 units b. 24-30 units, 8-12 units

b. 24-30 units, 8-12 units. Dextrins are polymers of glucose intermediate between the disaccharides like maltose, and the large polymers like starch. It is produced by enzymatic hydrolysis of starch (not glycogen). Maltoses are the penultimate product of the breakdown of starch before it goes to glucose; glycogen would not produce maltose. The complete hydrolysis of starch by amylases to form glucose goes like this: Starch-->Dextrins-->Maltose--> Glucose.

An enzyme used in both glycolysis & gluconeogenesis is: a. Phosphofructokinase-1 b. 3-phosphoglycerate kinase c. Hexokinase d. Phosphoglucose isomerase

b. 3-phosphoglycerate kinase. Phosphoglycerate kinase (PGK 1) is an enzyme that catalyzes the reversible transfer of a phosphate group from 1,3-bisphosphoglycerate (1,3-BPG) to ADP producing 3-phosphoglycerate (3-PG) and ATP. Like all kinases it is a transferase. The β-D-anomer of glucose is catabolized by hexokinase more efficiently than the α-D-anomer.

D-Erythrose & D-Threose is an example of? a. 3-carbon D-Aldoses b. 4-carbon D-Aldoses c. 5-carbon D-Aldoses d. 6-carbon D-Aldoses

b. 4-carbon D-Aldoses.

D-Erythrulose is a a. 3-carbon D-Ketose b. 4-carbon D-Ketose c. 5-carbon D-Ketose d. 6-carbon D-Ketose

b. 4-carbon D-Ketose.

What's the water content of the lungs? a. 80-85% b. 75-80% c. 70-75% d. 8-10%

b. 75-80%

What is the common fate of these amino acids: Isoleucine, Leucine & Tryptophan? a. Acetoacetyl CoA b. Acetyl-CoA c. Pyruvate

b. Acetyl-CoA

In precursor biosynthesis, which is a major area of carbon metabolism in which the Krebs Cycle is involved, which of the precursors is responsible for the production of glutamate? a. Citrate b. Alpha-Ketoglutarate c. Succinyl-CoA d. Malate

b. Alpha-Ketoglutarate. The Glutamate can then be used to make amino acids and purines. The reverse reaction, in which glutamate is converted to alpha-ketoglutarate, is possible by a transamination reaction.

When a protein is hydrolyzed there is: a. A decrease in free carboxyl groups b. An increase in free amino groups c. A large decrease in pH d. Formation of peptide bonds

b. An increase in free amino groups. Proteins are just condensed chains of amino acid monomers. They are formed by bonds between the amino group of one AA and the carboxyl group of another. Hydrolysis of these proteins would release said amino acids and free up both ends. Proteins are formed by condensation reactions amongst the amino groups and carboxyl groups of individual amino acids. Hydrolysis of our proteins would free up those amine and carboxyl groups to react with other molecules.

What term is defined as a condition in which the blood is deficient in red blood cells, in haemoglobin, or in total volume? a. Nephrotic Syndrome b. Anaemia c. Pentose Phosphate Pathway d. Lean Body Mass

b. Anaemia. Anaemia is a deficiency in the number or quality of red blood cells in your body. Red blood cells carry oxygen around your body using a particular protein called haemoglobin. Anaemia means that either the level of red blood cells or the level of haemoglobin is lower than normal.

Which of the following glucogenic amino acids form Glutamate? a. Alanine, Glycine, Threonine, Cysteine & Serine b. Arginine, Proline, Histidine & Glutamine c. Asparagine & Aspartate d. Tyrosine & Phenylamine

b. Arginine, Proline, Histidine & Glutamine. That glutamate can then be fed into the citric acid cycle by being converted to alpha-ketoglutarate. This reaction is catalyzed by glutamate dehydrogenase.

Which of the following amino acids is the precursor for Creatine, Urea & Nitric Oxide? a. Glutamate b. Arginine c. Glycine d. Phenylalanine

b. Arginine.

Proteins are characterized by their a. Rapid diffusion rates compared to salts b. Behaviour as amphoteric dipolar ions c. Lack of specific molecular configuration d. Great stability under various physical conditions

b. Behaviour as amphoteric dipolar ions. Proteins are made up of amino acids which are themselves amphoteric (act as acids and bases), and can be zwitterions (have both + and (-) charge in solution). Aka dipolar ions. Proteins have very slow diffusion rates (they're very big remember). Proteins are some of the most complex biological structures. All of the proteins in the body are formed from combinations of 20 amino acids into specific combinations, and further specific folding, twisting, bonding and joining of chains up to the quaternary structure. They are very specific in their molecular configuration.

What is characteristic of the secondary structure of proteins? a. Amino acid sequence b. Beta-sheets and alpha-helix c. 3-D folding d. Interaction of enzyme with protein

b. Beta-sheets and alpha-helix The secondary structure refers to local folded structures that form within a polypeptide due to interactions between atoms of the backbone. The backbone just is the polypeptide chain apart from the R groups;i.e. the secondary structure does not involve R group atoms.

The most common ketoses are those with their ketone function at a. C1 b. C2 c. C3 d. C4

b. C2. The most common ketoses are dihydroxyacetone, ribulose, and fructose. The hydroxyl and either the aldehyde or ketone functions of monosaccharides react intermolecularly to form cyclic hemiacetals and hemiketals, and these cyclic sugars have 1 of 2 anomeric carbons. The anomeric carbon is the carbon of the carbonyl group that results from the cyclization of the sugar, forming a chiral center.

Which enzyme, in preparation for beta oxidation, forms acylcarnitine by transferring the acyl group to carnitine? a. Acyl-CoA synthetase b. Carnitine acyltransferase 1 c. Translocase d. Carnitine acyltransferase 2

b. Carnitine acyltransferase 1. CoA is released in the process.

What is the "killer enzyme" that plays a role in apoptosis, or programmed cell death? a. AMP-activated protein kinase b. Caspase

b. Caspase. Caspases are a family of protease enzymes playing essential roles in programmed cell death (including apoptosis, pyroptosis and necroptosis) and inflammation.

What is true about enzyme activity? a. Lysozyme forms covalent bonds with its substrate and not chymotrypsin b. Chymotrypsin forms covalent bonds with its substrate and not lysozyme c. Both form covalent bonds with substrates

b. Chymotrypsin forms covalent bonds with its substrate and not lysozyme. Chymotrypsin operates through a particular type of ping-pong mechanism called covalent hydrolysis. This means that the enzyme first forms a covalent bond with the target substrate, displacing the more stable moiety into solution. This enzyme-substrate complex is called the enzyme intermediate.

In the electron transport chain, Succinate dehydrogenase (Succinate to Ubiquinone) is a. Complex 1 b. Complex 2 c. Complex 3 d. Complex 4

b. Complex 2. This enzyme is a flavoprotein located on the outer face of the inner mitochondrial membrane, and like acyl CoA dehydrogenase it channels electrons into the respiratory chain by reducing ubiquinone.

Serum isoenzyme measurements are useful in diagnosis because: a. The composition of the serum changes specifically with specific diseases b. Different tissues may contain different isoenzymes and any damage to a particular tissue releases only its characteristic isoenzymes into serum c. Different tissues may have the same isoenzyme composition for a given disease d. The disease process causes the damaged tissue to synthesize and release specific isoenzymes as a defense mechanism for a given disease

b. Different tissues may contain different isoenzymes and any damage to a particular tissue releases only its characteristic isoenzymes into serum. LDH is one of the most commonly measured isoenzymes and it has 5 forms which are specific to certain tissues in the body such as the heart, blood vessels, brain, kidney and lungs.Elevated levels of a particular isoenzyme can help diagnose where in the body tissues have been damaged (e.g in a heart attack).

Which of the following is an excess fluid that leaks around the cells of the capillaries? a. Transudate b. Exudate

b. Exudate. Inflammation is the most common cause of this type of fluid. The fluid is exudate if the effusion protein/serum protein ratio is greater than 0.5, effusion LDH/serum LDH ratio is greater than 0.6 and the effusion LDH level > two thirds the upper limit of the lab's range of serum LDH. The common causes of this type of fluid include infections and malignant processes.

The mechanism of metabolism of triacylglycerols is facilitated by a group of enzymes called lipases. Which hormone activates hormone-sensitive lipases? a. Insulin b. Glucagon c. Testosterone d. Oestrogen

b. Glucagon. Its name was coined to reflect the ability of hormones such as catecholamines, ACTH, and glucagon to stimulate the activity of this intracellular neutral lipase. Hormonal activation of HSL occurs via cyclic AMP dependent protein kinase (PKA), which phosphorylates HSL.

What term is given to the presence of both normal and mutated mtDNA? a. Selectively Permeable b. Heteroplasmy c. Apoptosis d. Cytoskeleton

b. Heteroplasmy; Heteroplasmy is the presence of more than one type of organellar genome (mitochondrial DNA or plastid DNA) within a cell or individual. It is an important factor in considering the severity of mitochondrial diseases. It results in variable expression in mitochondrial inherited disease.

Which of the following amino acids is the precursor for Histamine? a. Tryptophan b. Histidine c. Glycine d. Phenylalanine

b. Histidine.

What type of oedema leads o cellular damage, which will stimulate the release of mediators and the inflammatory response? a. Interstitial oedema b. Intracellular oedema

b. Intracellular oedema; It occurs due to hypoxic injury, when the blood flow falls below a certain critical level required to maintain cell membrane viability.

Transamination is an exchange of functional groups between any amino acid and an α-keto acid. Which amino acids, however, are not transaminated? a. Leucine & Lysine b. Lysine, proline, and threonine c. d.

b. Lysine, proline, and threonine. The amino group is usually transferred to the keto carbon atom of pyruvate, oxaloacetate, or α-ketoglutarate, converting the α-keto acid to alanine, aspartate, or glutamate, respectively. Transamination reactions are catalyzed by specific transaminases (also called aminotransferases), which require pyridoxal phosphate as a coenzyme.

PEP undergoes the reversal of glycolysis until fructose 1,6-bisphosphate is produced. The conversion of fructose 1,6-bisphosphate to fructose 6-phosphate by the enzyme fructose 1,6-bisphosphatase requires which of the following? a. Water b. Mg 2+ ions c. Na+ ions d. Calcium ions

b. Mg 2+ ions.

Glucose 6 phosphate is converted to Fructose 6 phosphate by the enzyme phosphohexose isomerase in the presence of: a. Water b. Mg 2+ c. Na+ d. Calcium

b. Mg 2+. Since glucose 6-phosphate is also a product of gluconeogenesis, it serves as a substrate for glucose-6-phosphatase in the liver. The action of this enzyme releases free glucose into the bloodstream.

Which of the following is a chemical species with an affinity for a positive charge? a. Electrophile b. Nucleophile

b. Nucleophile. A nucleophilic attack is when an electron rich neucleophile attacks a positive of partially positive electrophile.

Which of the following is the most common urea cycle disorder resulting due to a mutated and ineffective form of the enzyme. a. Argininosuccinate synthase deficiency b. Ornithine transcarbamylase deficiency c. Carbamoyl Phosphate Synthetase deficiency

b. Ornithine transcarbamylase (OTC) deficiency. It's an X-linked recessive disorder caused by different mutations in the OTC gene; thus males are more seriously affected. Complications may include mental retardation and developmental delay.

In glycolysis, which enzyme adds a phosphate group to fructose 6-phosphate to produce fructose-1-6-bisphosphate? a. glucose-6-phosphate isomerase b. Phosphofructokinase-1 c. Glyceraldehyde-3-phosphate dehydrogenase

b. Phosphofructokinase-1. PFK-1 is promoted by AMP and fructose-2,6-bisphosphate. It's inhibited by ATP and Citrate.

What enzyme converts pyruvate to acetyl-CoA, and thereby increases the influx of acetyl-CoA from glycolysis into the citric acid cycle? a. Fatty acid synthetase b. Pyruvate dehydrogenase c. Acetyl CoA carboxylase d. Protein kinase

b. Pyruvate dehydrogenase. Pyruvate dehydrogenase (PDH) is a convergence point in the regulation of the metabolic finetuning between glucose and FA oxidation.

Amino acid catabolism is most likely to occur when a person is starving or fasting. What precursor of gluconeogenesis is obtained primarily from amino acid catabolism? a. Lactate b. Pyruvate c. Glycerol

b. Pyruvate. The precursors of gluconeogenesis are lactate, glycerol, amino acids, and with propionate making a minor contribution. The gluconeogenesis pathway consumes ATP, which is derived primarily from the oxidation of fatty acids.

What's the product of the pentose phosphate pathway? a. Glucose-6-phosphate b. Ribose-5-phosphate c. Pyruvate

b. Ribose-5-phosphate.

Which of the following refers to particularly stable arrangements of amino acids residues in a protein that give rise to recurring patterns? a. Primary structure b. Secondary structure c. Tertiary structure d. Quaternary structure

b. Secondary structure (regular arrangement)

Which of the following is most appropriate regarding action potential? a. Multiple depolarizing events minimizes the chance of action potential generation. b. Threshold event generates action potential. c. Subthreshold cannot stimulate a postsynaptic membrane. d. Depolarized synaptic membrane is more negative than hyperpolarized membrane. e. Subthreshold is enough to generate an action potential.

b. Threshold event generates action potential. The action potential is an explosion of electrical activity that is created by a depolarizing current. This means that some event (a stimulus) causes the resting potential to move toward 0 mV. When the depolarization reaches about -55 mV a neuron will fire an action potential. This is the threshold.

What is the principal physiological function of bile salts? a. To permit cholesterol catabolism b. To catalyze lipid digestion c. To permit gastrointestinal absorption of dietary lipids d. To control enterohepatic circulation

b. To catalyze lipid digestion. Their main function is in the emulsification of fats. Bile salts are really for the breakdown of lipids; once they're all broken down they can be easily absorbed in the small intestines. Bile salts in and of themselves form part of the enterohepatic circulation of bile from the liver to the intestines to aid in digestion and back again-- but they don't control it.

Which of the following represents a protein modification which changes enzyme activity in a non-reversible manner? a. Competitive inhibition b. Zymogen activation c. Phosphorylation of a serine residue of the protein d. Coenzyme activation

b. Zymogen activation. Zymogens are inactive forms of enzymes that have portions blocking the active site. Cleaving these portions from the enzyme, results in its activation. Naturally, this is not a reversible process. One major example is trypsinogen which is cleaved to form trypsin.

Glucose is a (an): a. oligosaccharide b. aldohexose c. aldopentose d. ketohexose

b. aldohexose. Glucose is a 6-carbon sugar (hexose) with an aldehyde group (aldose). Hence it is an aldohexose.

The predominant structural feature in myosin molecules is: a) a beta structure. b) an alpha helix. c) the Fab domain. d) the light chain.

b. an alpha helix. Myosin is a special protein that converts adenosine triphosphate (ATP), a molecule that cells use in order to live and work, into mechanical energy (energy for work). This will then generate force and movement. It works closely with a globular protein called actin that polymerizes to create actin filaments.

About 40g of urea is excreted per day. In the urea cycle, which enzyme combines arginine with water to re-form ornithine; releasing urea in the process? a. glutamate dehydrogenase b. arginase c. argininosuccinate lyase (or argininosuccinase) d. argininosuccinate synthetase

b. arginase. This enzyme is present only in liver of ureotelic animals. Arginine is hydrolyzed to generate urea and ornithine. Ornithine is transported back into the mitochondrion to begin another cycle

In the urea cycle, what enzyme catalyzes the production of carbamoyl phosphate from ammonia? a. glutamate dehydrogenase b. carbamoyl phosphate synthetase c. ornithine transcarbamylase d. argininosuccinate synthetase

b. carbamoyl phosphate synthetase. Therefore, the urea cycle begins with the coupling of free NH3 with HCO3 to form carbamoyl phosphate. This reaction occurs in the mitochondria. It requires 2 ATP and bicarbonate ion.

During muscle contraction, hydrolysis of ATP results in a change in the: a) confirmation of actin. b) conformation of myosin. c) structure of the myofibrils. d) structure of the sarcoplasmic reticulum.

b. conformation of myosin. The motion of muscle shortening occurs as myosin heads bind to actin and pull the actin inwards. This action requires energy, which is provided by ATP. Myosin binds to actin at a binding site on the globular actin protein.

In beta oxidation, Enoyl-CoA is hydrated to hydroxyacyl-CoA via a. acyl-coa dehydrogenase b. enoyl-coa hydratase c. hydroxyacyl-CoA dehydrogenase d. ketothiolase

b. enoyl-coa hydratase.

Which of the following polysaccharides is NOT a polymer of glucose? a. amylose b. inulin c. glycogen d. amylopectin

b. insulin. Inulin is a polysaccharide made up of FRUCTOSE monomers and not glucose. Amylopectin forms 80% of starch and is a glucose polymer that branches every 24-30 glucose units with alpha-1,6-glycosidic bonds at each branch. Amylose is a glucose polymer. It is made up of long and unbranched glucose units joined by alpha-1,4-glycosidic (sugar) linkages. Fun fact: It is amylose that makes water cloudy when boiling potatoes. Amylose forms 20% of starch, while amylopectin forms the other 80%.

What term is used to define an enlargement of the right ventricle, resulting from the chronic disease within the lungs, that causes congestion within the pulmonary circulation and resistance of blood flow to the lungs? a. Prostaglandins b. Right Ventricular Failure c. Cardiomegaly d. Embolism

b. right ventricular failure. Failure of the Right Ventricle results in systemic venous hypertension, and can lead to the following signs/symptoms: - Peripheral edema. - Anorexia, nausea, and abdominal pain related to congestion hepatomegaly. - Fatigue, dypnea (related to inadequate Cardiac Output)

Each of the following interfere with collagen crosslinking by affecting lysyl oxidase EXCEPT: a. copper deficiency b. vitamin D deficiency c. nitrile inhibitors d. lathyrus odoratus (sweet peas)

b. vitamin D deficiency. Vitamin D deficiency has actually been shown to increase the activity of lysyl oxidase in baby chicks. Lysyl oxidase is indeed a copper-dependent enzyme. But if you had a copper deficiency, lysyl oxidase would be affected and the last stage of collagen formation would be affected (crosslinking).

Ketones oxidize to produce energy for the brain. In ketone oxidation, __________________ is oxidized to acetoacetate and NADH is released.

beta (β)-hydroxybutyrate. An HS-CoA molecule is added to acetoacetate, forming acetoacetyl CoA. The carbon within the acetoacetyl CoA that is not bonded to the CoA then detaches, splitting the molecule in two. This carbon then attaches to another free HS-CoA, resulting in two acetyl CoA molecules. These two acetyl CoA molecules are then processed through the Krebs cycle to generate energy.

Most diet lipids of mammals are Triacylglycerols (TGs). In the small intestine, fat particles are coated with ________ ___________ and digested by pancreatic lipases.

bile salts. Lipases degrade TGs to free fatty acids and a 2-monoacylglycerol. Lipase catalyzes hydrolysis at the C-1 and C-3 positions of a TG.

Free fatty acids in blood are: a) metabolically inert b) bound mainly to β-lipoproteins c) stored in fatty deposits d) bound mainly to serum albumin

bound mainly to serum albumin. Free fatty acids are the major fat fuel in the body, and when they are elevated in the blood they are thought to raise the risk of cardiovascular disease by causing insulin resistance (in some cases leading to diabetes), raising blood pressure, and other effects.

During prolonged starvation, brain metabolises which of the following? a) Glucose b) Fatty acyl carnitine c) Acetoacetate d) Free fatty acids

c) Acetoacetate. During starvation, most tissues utilise fatty acids and/or ketone bodies to spare glucose for the brain. Glucose utilisation by the brain is decreased during prolonged starvation as the brain utilize ketone bodies as the major fuel. High concentrations of ketone bodies result in significant excretion of ketones. Acetoacetate, beta-hydroxybutyrate, and their spontaneous breakdown product, acetone, are known as ketone bodies. The ketone bodies are released by the liver into the blood.

The deficiency of which of the following enzyme causes Citrullinemia- a condition with an accumulation of citrulline in the blood a) Carbamoyl Phosphate I b) Ornithine Transcarbamylase c) Argininosuccinate synthase d) Arginosuccinase

c) Argininosuccinate synthase

Which of the following lipid is mostly present in mitochondrial membranes? a) Lecithin b) Cephalin c) Cardiolipin d) Ceramide

c) Cardiolipin. Cardiolipin is a unique phospholipid, which is almost exclusively localized in the mitochondrial inner membrane where it is synthesized from phosphatidylglycerol and cytidine diphosphate-diacylglycerol. After primary synthesis, the mature acyl chain composition of cardiolipin is achieved by at least two remodeling mechanisms. In the mitochondrial membrane cardiolipin plays an important role in energy metabolism, mainly by providing stability for the individual enzymes and enzyme complexes involved in energy production.

Which enzyme, in the 6th step of glycolysis, catalyzes the phosphorylation and oxidation of GAP, yielding the 1st high-energy intermediate of the pathway, 1,3-biphosphoglycerate (1,3-BPG)? a) Hexokinase b) Aldolase c) Glyceraldehyde-3-phosphate dehydrogenase d) Triosephosphate isomerase

c) Glyceraldehyde-3-phosphate dehydrogenase (GAPDH). This phosphorylation produces 2 (because there are 2 molecules of 3-carbon GAP) NADH from NAD+ and Pi. The enzyme features an active-site cysteine sulfhydryl group that allows for the direct hydride transfer of a proton from C1 of GAP to NAD+ by acting as a nucleophile during its attack on the aldehyde. This forms a thiohemiacetal intermediate that undergoes oxidation to an acyl thioester during the direct hydride transfer to NAD+. The thioester intermediate then undergoes a nucleophilic attack by Pi to form 1,3-BPG, a mixed anhydride.

Familial hypercholesterolemia is a genetic disorder of lipid metabolism. The defect lies in a) Transport of cholesterol from extrahepatic tissue to the liver b) Impairment of cholesterol degradative pathway c) Impairment of uptake of cholesterol by tissues d) Impairment of HDL metabolism due to deficiency of Apo-A

c) Impairment of uptake of cholesterol by tissues

Fructose is a (an): a) Oligosaccharide b) Aldohexose c) Ketohexose d) aldopentose

c) Ketohexose. A molecule, of fructose and glucose linked together form one molecule of sucrose. Fructose is a simple, single unit, Ketohexose, sugar called a monosaccharide.

The regulation of oxidative phosphorylation depends on a) Magnitude of ion motive force b) Magnitude of electron motive force c) Magnitude of proton motive force d) None

c) Magnitude of proton motive force. There are a lot of different ways organisms acquire food. Almost all aerobic organisms (organisms that require oxygen to live) use oxidative phosphorylation, in one way or another, to produce the basic energy currency of the cell needs to function: ATP (adenosine triphosphate). Oxidative phosphorylation is the fourth step of cellular respiration, and produces the most of the energy in cellular respiration.

Where does oxidative phosphorylation take place? a) Ribosomes b) Nucleus c) Mitochondria d) Cell membrane

c) Mitochondria. Oxidative phosphorylation takes place in the inner mitochondrial membrane, in contrast with most of the reactions of the citric acid cycle and fatty acid oxidation, which take place in the matrix.

Active transport includes the following features EXCEPT: a) Influx of K+ and efflux of Na+ b) Hydrolysis of ATP c) No specific carrier is required d) Moves the molecules against a concentration gradient.

c) No specific carrier is required

The enzymes of fatty acid synthesis are packaged together in a complex called as fatty acid synthase (FAS). What's the product of FAS action? a) Arachidonic acid b) Oleic acid c) Palmitic acid d) Stearic acid

c) Palmitic acid. Modifications of this primary FA leads to other longer (and shorter) FA and unsaturated FA. The fatty acid molecule is synthesized 2 carbons at a time. FA synthesis begins from the methyl end and proceeds toward the carboxylic acid end. Thus, C16 and C15 are added first and C2 and C1 are added last.

In metabolism, NAD is involved in a) Spontaneous reaction b) Elimination reaction c) Redox reactions d) None

c) Redox reactions. In metabolism, nicotinamide adenine dinucleotide is involved in redox reactions, carrying electrons from one reaction to another. The cofactor is, therefore, found in two forms in cells: NAD+ is an oxidizing agent - it accepts electrons from other molecules and becomes reduced.

Which of the following is not the effect of cholecystokinin released from mucosal cells of jejunum? a) Contraction of gall bladder and release of bile acids b) The release of digestive enzymes from the exocrine pancreas c) The slower release of gastric content into the stomach d) Increased of gastric motility

c) The slower release of gastric content into the stomach

Which of the following is simple lipid? a) Lecithin b) Fatty acid c) Triacylglycerol d) Steroids

c) Triacylglycerol. A triglyceride (TG, triacylglycerol, TAG, or triacylglycerol) is an ester derived from glycerol and three fatty acids (from tri- and glyceride). Triglycerides are the main constituents of body fat in humans and other vertebrates, as well as vegetable fat.

In Complex I (NADH ubiquinone oxireductase, Type I NADH dehydrogenase) how many electrons are removed from NADH and ultimately transferred to a lipid-soluble carrier, ubiquinone (UQ)? a. 10 b. 4 c. 2 d. 0

c. 2 The reduced product, ubiquinol (UQH2), freely diffuses within the membrane, and Complex I translocates four protons (H+) across the membrane, thus producing a proton gradient. Complex I is one of the main sites at which premature electron leakage to oxygen occurs, thus being one of the main sites of production of superoxide.

What is characteristic of the tertiary structure of proteins? a. Amino acid sequence b. Beta-sheets and alpha-helix c. 3-D folding d. Interaction of enzyme with protein

c. 3-D folding; The overall three-dimensional structure of a polypeptide is called its tertiary structure. The tertiary structure is primarily due to interactions between the R groups of the amino acids that make up the protein.

Normally ________ of the body's phenylalanine is converted into tyrosine. a. 1/4 b. 2/4 c. 3/4 d. 4/4

c. 3/4. The rest becomes incorporated into proteins. Because the major outflow pathway is blocked in PKU, the blood level of phenylalanine is elevated 20 fold. The excess phenylalanine spills over into the urine.

D-Lyxose is an example of? a. 3-carbon D-Aldoses b. 4-carbon D-Aldoses c. 5-carbon D-Aldoses d. 6-carbon D-Aldoses

c. 5-carbon D-Aldoses.

D-Ribose, D-Arabinose & D-Xylose are examples of? a. 3-carbon D-Aldoses b. 4-carbon D-Aldoses c. 5-carbon D-Aldoses d. 6-carbon D-Aldoses

c. 5-carbon D-Aldoses.

D-Ribulose & D-Xylulose are examples of a. 3-carbon D-Ketoses b. 4-carbon D-Ketoses c. 5-carbon D-Ketoses d. 6-carbon D-Ketoses

c. 5-carbon D-Ketoses.

The reaction catalyzed by phosphofructokinase-1 (PFK-1) is? a) Inhibited by Fructose 2,6 bisphosphate b) Activated by a high concentration of ATP c) A regulatory step of glycolysis d) All of the above

c. A regulatory step of glycolysis. In reactions 1-5, which consume 2 ATP, glucose is phosphorylated and split to yield 2 triose molecules of glyceraldehyde-3-phosphate (GAP). In reactions 6-10, which generate 4 ATP, 2 molecules of GAP are enzymatically converted to pyruvate. The net production of ATP for the pathway is 2 ATP produced per glucose molecule catabolized.

What is an allosteric effector of an enzymic reaction? a. A competitive inhibitor b. An uncompetitive inhibitor c. A substance which affects enzymic activity by binding to a site other than the substrate binding site d. A denaturant of the enzyme

c. A substance which affects enzymic activity by binding to a site other than the substrate binding site. Allosteric effectors modulate enzyme activity. They don't bind to the active site so they could not be competitive inhibitors. Allosteric enzymes work by binding to allosteric sites to either increase (+) or decrease (-) enzyme activity. They do not denature enzymes.

Which of the following is the rate determining enzyme of fatty acid synthesis? a. Fatty acid synthetase b. Pyruvate dehydrogenase c. Acetyl CoA carboxylase d. Protein kinase

c. Acetyl CoA carboxylase The rate-limiting step in the fatty acid synthesis is the enzyme acetyl-CoA carboxylase, which uses acetyl-CoA to make malonyl-CoA. This is the only enzyme regulated in fatty acid synthesis. This reaction requires ATP.

_________________ is the carrier of both ammonia and the carbon skeleton of pyruvate from muscle to liver. a. Glutamate b. Glutamine c. Alanine

c. Alanine. The ammonia is excreted and the pyruvate is used to produce glucose which is returned to the muscle. Increased concentrations of ammonia in the blood and other biological fluids leads to the diffusion of a ammonia across the blood brain barrier. This results in the synthesis of glutamate from alpha-ketoglutarate; thus increasing the synthesis of glutamine.

What term is used to define the process of programmed cell death that may occur in multicellular organisms? a. Selectively Permeable b. Heteroplasmy c. Apoptosis d. Cytoskeleton

c. Apoptosis; it's essential for maintaining cell balance; it only occurs when given certain signals from cells

Which of the following glucogenic amino acids form Oxaloacetate? a. Alanine, Glycine, Threonine, Cysteine & Serine b. Arginine, Proline, Histidine & Glutamine c. Asparagine & Aspartate d. Tyrosine & Phenylamine

c. Asparagine & Aspartate.

Cholesterol is metabolized in the mammalian body primary by conversion to: a. Vitamin D b. Fatty acids c. Bile acids d. Carbon dioxide

c. Bile acids. Failure of bile acids/their conjugated salts to keep cholesterol in solution results in the formation of gallstones in the gallbladder.

Which of the following is an autosomal recessive metabolic disorder that is treated with Benzoate and Phenylacetate? a. Argininosuccinate synthase deficiency b. Ornithine transcarbamylase deficiency c. Carbamoyl Phosphate Synthetase deficiency

c. Carbamoyl Phosphate Synthetase 1 (CPS1) deficiency. It is an autosomal recessive metabolic disorder associated with mental retardation and developmental delay.

What term is defined as the abnormal enlargement of the heart? a. Prostaglandins b. Right Ventricular Failure c. Cardiomegaly d. Embolism

c. Cardiomegaly. It is frequently associated with heart failure as the heart enlarges in an effort to compensate for its decreased pumping ability.

In the electron transport chain, Cytochrome bc1 complex is a. Complex 1 b. Complex 2 c. Complex 3 d. Complex 4

c. Complex 3. Both complex I and complex II pass their electrons to a small, mobile electron carrier called ubiquinone (UQ), which is reduced to form UQH2 and travels through the membrane, delivering the electrons to complex III.

For fatty acids, the storage form is triacylglycerol (TAG). What precursor of gluconeogenesis is obtained from triacylglycerol catabolism? a. Lactate b. Pyruvate c. Glycerol

c. Glycerol. In humans, the main gluconeogenic precursors are lactate, glycerol (which is a part of the triacylglycerol molecule), alanine and glutamine. Altogether, they account for over 90% of the overall gluconeogenesis.

Which of the following amino acids is the precursor for Heme? a. Tryptophan b. Histidine c. Glycine d. Phenylalanine

c. Glycine.

Which of the following is NOT optically active? a. Leucine b. Alanine c. Glycine d. Lysine

c. Glycine. Glycine is the only amino acid that is achiral (and hence not optically active). Every other amino acid has four different groups attached to the alpha carbon and is automatically chiral.

What's the simplest amino acid? a. Methionine b. Histidine c. Glycine d. Proline

c. Glycine. The simplest of the amino acids, glycine, has just H as an R-group. Amino acids are the structural elements from which proteins are built. When amino acids bond to each other, it is done in the form of an amide , making a connection which is called a peptide linkage.

In the very first step of glycolysis, which enzyme transfers a phosphoryl group from ATP onto the 6th carbon of glucose to form Glucose-6-phosphate (G6P). a. Phosphofructokinase-1 b. 3-phosphoglycerate kinase c. Hexokinase d. Phosphoglucose isomerase

c. Hexokinase. A kinase is an enzyme that transfers phosphoryl groups between ATP and a substrate. Hexokinase is a nonspecific enzyme that works on a variety of hexoses, such as D-glucose, D-mannose, and D-fructose.

The class of lipoproteins that is beneficial to atherosclerosis is: a) Low-density of lipoproteins b) Very low-density lipoproteins c) High-density lipoproteins d) Chylomicrons

c. High-density lipoproteins. High-density lipoprotein (HDL) cholesterol is a lipoprotein that contains a small amount of cholesterol and carries cholesterol away from body cells and tissues; therefore, high levels are desirable, ≥ 60 mg/dl.

A sulfur-containing amino acid is: a. Sulfinylpyruvic acid b. Chondroitin sulfate c. Homocysteine d. Tryptophan

c. Homocysteine. Homocysteine is just cysteine with a methylene bridge (and cysteine you will recall has a sulfur group).Why should you remember homocysteine? Well it's a part of B12 and folate metabolism as seen in basic haematology. High levels of homocysteine are usually associated with low levels of B6, B12 and folate, which are risk factors for megaloblastic anemia and heart disease.

The degree of inhibition caused by a competitive inhibitor can be decreased by a. Adding water, thus diluting out the EI complex b. Increasing the enzyme concentration c. Increasing the substrate concentration d. Increasing both the enzyme and inhibition concentration

c. Increasing the substrate concentration. Since this inhibitor is competitive, increasing the substrate concentration will eventually allow the substrate to overpower the inhibitor and reduce its inhibitory effect.

Which of the following is not correct concerning the cooperative binding of a ligand to a protein? a) It is usually a form of allosteric interaction. b) It is usually associated with proteins with multiple subunits. c) It rarely occurs in enzymes. d) It results in a nonlinear Hill Plot.

c. It rarely occurs in enzymes. If the binding of a ligand at one site increases the affinity for the ligand at another site, the macromolecule exhibits positive cooperativity. Conversely, if the binding of a ligand at one site lowers the affinity for the ligand at another site, the protein exhibits negative cooperativity.

An important enzyme for the synthesis of glucose from lactate is also a useful serum indicator of tissue damage (e.g myocardial infarct). That enzyme is: a. Creatinine kinase b. Glutamate-oxaloacetate transaminase c. Lactate dehydrogenase d. Pyruvate dehydrogenase

c. Lactate dehydrogenase. LDH is leaked into the blood following tissue damage but it also helps convert lactate to glucose through a process called the Cori Cycle. Creatinine kinase is a great enzyme for detecting tissue damage-- especially in the case of heart attack through its isozyme CK-MB. However, it is not involved in the Cori cycle and the synthesis of glucose from lactate.

Which of the following has the greatest blood flow per 100mg of tissue? a. Brain b. Heart Muscle c. Liver d. Kidney

c. Liver. Distribution of cardiac output (CO) liver > kidney > muscles > brain The liver receives the highest percentage of CO because kidneys are much smaller than the liver, kidneys have the highest percentage of CO per gram of tissue.

Which one of the following membranes contains the highest percentage of lipid? a. Endoplasmic reticulum b. Erythrocyte plasma membrane c. Myelin sheath d. Inner mitochondrial membrane

c. Myelin sheath. Myelin sheaths in the CNS have a whopping 70% lipids in their membranes while those in the PNS have closer to 80%. The ER membrane tends to be closer to the normal cell membrane in its lipid composition: ~ 50% protein and lipid. The red blood cells membranes have about equal lipids as proteins (50%). The inner mitochondrial membrane is only around 20% lipid with the rest of the membrane being mostly proteins. These proteins are needed for things like the electron transport chain.The key take away here is that the % of lipids, and as a consequence the ratio of lipids:proteins in the different membranes varies according to function.

Which one of the following statements about aromatic amino acids is correct? a. All are strongly hydrophilic b. HIstidine's ring structure results in it being categorized as aromatic or basic, depending on pH. c. On a molar basis, tryptophan absorbs more ultraviolet light than tyrosine. d. The major contribution to the characteristic absorption of light at 280nm by proteins is the phenylalanine R group.

c. On a molar basis, tryptophan absorbs more ultraviolet light than tyrosine.

What term is defined as a metabolic process that produces NADPH and ribose 5-phosphate for nucleotide synthesis? a. Nephrotic Syndrome b. Anaemia c. Pentose Phosphate Pathway d. Lean Body Mass

c. Pentose Phosphate Pathway. The pentose phosphate pathway (also called the phosphogluconate pathway and the hexose monophosphate shunt) is a metabolic pathway parallel to glycolysis. It generates NADPH and pentoses (5-carbon sugars) as well as ribose 5-phosphate, a precursor for the synthesis of Nucleotides.

In Phenylketonuria, plasma phenylalanine concentration rises after ingestion of phenylalanine. What is the enzyme that is absent or deficient in Phenylketonuria? a. AMP-activated protein kinase b. Caspase c. Phenylalanine Hydroxylase

c. Phenylalanine Hydroxylase. Thus, phenylalanine accumulates in all body fluids as it will not be able to be converted to tyrosine. The degree of defect is variable but it is severe in the majority of the patients. Rare cases of PKU are caused by the absence of the defect in the conversion of GTP into tetrahydrobiopterin cofactor.

Protein transport to the interior of the cell generally occurs by which one of the following: a. Passive diffusion b. Carrier-mediated active transport c. Pinocytosis d. Exocytosis

c. Pinocytosis; Carrier-mediated transport in which the net movement is down a concentration gradient, and which is therefore passive, is called facilitated diffusion. Carrier-mediated transport that occurs against a concentration gradient, and which therefore requires metabolic energy, is called active transport.

In alcoholic fermentation, which enzyme catalyzes the conversion of Pyruvate to Acetaldehyde a. Lactate dehydrogenase b. Alcohol dehydrogenase c. Pyruvate dehydrogenase d. Pyruvate decarboxylase

c. Pyruvate dehydrogenase

What's the product of glycolysis? a. Glucose-6-phosphate b. Ribose-5-phosphate c. Pyruvate

c. Pyruvate.

What is the common fate of these amino acids: Alanine, Cysteine, Glycine, Serine, Thereonine & Tryptophan? a. Acetoacetyl CoA b. Acetyl-CoA c. Pyruvate

c. Pyruvate. In gluconeogenesis, Pyruvate reacts with carbon dioxide to form oxaloacetate. The enzyme pyruvate carboxylase catalyzes this reaction.

Which of the following characteristics is typical of the Fluid Mosaic Model of membrane structure? a. The proteins are all spread in a sheet over the surface of the lipid bilayer b. The proteins are all attached as spherical molecules to the surface of the membrane c. Some proteins have polypeptides consisting predominantly of nonpolar residues in the interior of the membrane d. All proteins are loosely attached to the exterior and interior surfaces of the lipid bilayer

c. Some proteins have polypeptides consisting predominantly of nonpolar residues in the interior of the membrane. For example: integral proteins

Which metabolite undergoes net oxidation in the TCA cycle? a. Oxaloacetate b. Malonate c. Succinate d. Fumarate

c. Succinate.

In precursor biosynthesis, which is a major area of carbon metabolism in which the Krebs Cycle is involved, which of the precursors is responsible for the production of heme & chlorophyll? a. Citrate b. Alpha-Ketoglutarate c. Succinyl-CoA d. Malate

c. Succinyl-CoA

What is the primary male sex hormone and anabolic steroid? a. Insulin b. Glucagon c. Testosterone d. Oestrogen

c. Testosterone, In male humans, testosterone plays a key role in the development of male reproductive tissues such as testes and prostate, as well as promoting secondary sexual characteristics such as increased muscle and bone mass, and the growth of body hair.

In the steady state derivation of the Michaelis-Menten equation, each of the following statements is considered to be true EXCEPT which one? a. The concentration of enzymes-substrate complex is considered to be constant b. The concentration of free enzymes is considered to be constant c. The Michaelis constant (Km) is equal to the dissociation constant of the enzyme substrate complex d. The concentration of enzyme is much less than the concentration of substrate

c. The Michaelis constant (Km) is equal to the dissociation constant of the enzyme substrate complex. Where K1 catalyzes E+S-->ES and K2 catalyzes ES--> E + P. We define Km as the rate at which E is being reformed (K-1 + K2) over the rate at which it is being used up (K1). Here E represents the free enzyme and a (-) sign indicates a reverse reaction. So Km= (K2 +K-1)/K1.

In the regulation of an overall metabolic sequence, which of the following reactions is most likely to be modulated by allosteric effectors? a. Any reaction of the sequence b. The least reaction of the sequence c. The initial reaction of the sequence d. The fastest reaction of the sequence

c. The initial reaction of the sequence. The most common type of regulation we see in metabolism is negative feedback: whereby the products of a reaction can inhibit their own formation if they form in excess. Usually when present in excess, the end product of a series of steps will go back to inhibit the first step that resulted in its formation.

In preparation for beta oxidation, which enzyme can transport the resulting acylcarnitine into the mitochondrial matrix via the mitochondrial membrane, in exchange for carnitine? a. Acyl-CoA synthetase b. Carnitine acyltransferase 1 c. Translocase d. Carnitine acyltransferase 2

c. Translocase.

Which of the following is not an importance of body stores of lipid? a. Insulation from excess loss of heat b. Protection from mechanical trauma c. Use in fetal development d. Serve as an energy store in tissues of nutritional starvation

c. Use in fetal development.

A monoclonal antibody differs from a polyclonal antibody in that monoclonal antibodies: a) are labeled with chemicals that can be visualized. b) are produced by cells from the same organism that produced the antigen. c) are synthesized by a population of identical, or "cloned," cells. d) are synthesized only in living organisms.

c. are synthesized by a population of identical, or "cloned," cells. Monoclonal antibodies can have monovalent affinity, in that they bind to the same epitope (the part of an antigen that is recognized by the antibody). In contrast, polyclonal antibodies bind to multiple epitopes and are usually made by several different plasma cell (antibody secreting immune cell) lineages.

In the urea cycle, which enzyme cleaves argininosuccinate to form arginine and fumarate? a. glutamate dehydrogenase b. arginase c. argininosuccinate lyase (or argininosuccinase) d. argininosuccinate synthetase

c. argininosuccinate lyase (or argininosuccinase). The arginine continues in the cycle and the fumarate is given off. The fumarate enters the tricarboxylic acid cycle (Krebs Cycle). Therefore the urea cycle and the TCA cycle are linked.

Energy obtained through the transfer of electrons down the ETC is used to pump protons from the mitochondrial matrix into the intermembrane space, creating an electrochemical proton gradient (ΔpH) across the inner mitochondrial membrane. To which complex does ubiquinone pass its electrons? a. Complex 1 b. Complex 2 c. Complex 3 d. Complex 4

c. complex III (cytochrome bc1 complex), which passes them to cytochrome c (Cyt c). This proton gradient is largely but not exclusively responsible for the mitochondrial membrane potential (ΔΨM).[3] It allows ATP synthase to use the flow of H+ through the enzyme back into the matrix to generate ATP from adenosine diphosphate (ADP) and inorganic phosphate. Complex I (NADH coenzyme Q reductase) accepts electrons from the Krebs cycle electron carrier nicotinamide adenine dinucleotide (NADH) and passes them to coenzyme Q (ubiquinone), which also receives electrons from complex II (succinate dehydrogenase).

The end product of the acid hydrolysis of glycogen is: a. amylopectin b. dextrin c. glucose d. maltose

c. glucose The acid-catalyzed hydrolysis of glycogen to D-glucose is performed by amylases (which also break down starch). Phosphorylases will break down glycogen into Glucose-1-phosphate (G1P), which is converted to G6P to go into glycolysis.

The fundamental cause of sickle-cell disease is a change in the structure of: a) blood. b) capillaries. c) hemoglobin. d) red cells.

c. hemoglobin. Here, the most common problem is a combination sickle cell and beta thalassemia genes. The sickle cell mutation reflects a single change in the amino acid building blocks of the oxygen-transport protein, hemoglobin. This protein, which is the component that gives red cells their color, has two subunits.

In beta oxidation, Hydroxyacyl-CoA is oxidized to ketoacyl-CoA via a. acyl-coa dehydrogenase b. enoyl-coa hydratase c. hydroxyacyl-CoA dehydrogenase d. ketothiolase

c. hydroxyacyl-CoA dehydrogenase. During this process, NAD+ is converted to NADH and H+.

In the urea cycle, carbamoyl phosphate combines with ornithine and produces citrulline as a result of the dephosphorylation of carbamoyl phosphate. What enzyme is required for this step? a. glutamate dehydrogenase b. carbamoyl phosphate synthetase c. ornithine transcarbamylase d. argininosuccinate synthetase

c. ornithine transcarbamylase (OTC). Carbamoyl phosphate donates carbamoyl group to ornithine; producing citrulline in the process. This reaction takes place in the mitochondrial matrix. The formation of carbamoyl phosphate is a prerequisite for the urea cycle.

In hemoglobin, the transition from T state to R state (low to high affinity) is triggered by: a) Fe2+ binding. b) heme binding. c) oxygen binding. d) subunit association.

c. oxygen binding. The T to R transition requires that at least two of the haemoglobin subunits be bound by oxygen. Since haemoglobin in the T state only has a low affinity for oxygen, the conformational change can only occur under relatively high oxygen concentrations (such as in the lung capillaries). In the R state, haemoglobin binds to oxygen with much greater affinity, leading to any remaining deoxygenated subunits quickly binding to oxygen.

The terms "primary", "secondary" and "tertiary" structure of a protein refer to its: a. alpha-helix, beta-helix and gamma-helix b. peptide bonds and di-sulfide bonds c. peptide bonds, hydrogen bonds and non-covalent bonds d. electrostatic interactions, van der Waals forces and disulfide bonds

c. peptide bonds, hydrogen bonds and non-covalent bonds. The primary structure is made up of covalent bonds called peptide bonds, among individual amino acids. The secondary structures are made from twists and kinks due to hydrogen bonds. The tertiary structure includes a variety of non-covalent bonds and van der Waals forces (but also covalent disulfide bonds). The quaternary structure is formed from combining polypeptide chains.'

The major end product of protein-nitrogen metabolism in a man is: a. glycine b. uric acid c. urea d. ammonia

c. urea. Proteins are broken down to amino acids in the stomach and small intestines under the action of stomach enzymes and pancreatic enzymes. These amino acids are converted to the toxic ammonia for a short time, which enters the urea cycle to form urea. Urea can be safely excreted in the urine. The cycle also produces fumarate to be pumped into the Kreb's cycle but this is not a major end product.

Ammonia is very toxic to humans and especially to brain cells; thus, defects of the urea cycle causes hyperammonemia and can lead to brain damage. The urea cycle begins with the coupling of free NH3 with HCO3 to form what?

carbamoyl phosphate. This reaction takes place in the matrix of the liver mitochondria by the enzyme carbamoyl phosphate synthetase 1 (CPS1) which makes up 20 % of the protein of the mitochondrial matrix. The allosteric activator of CPS1 is N-acetylglutamic acid. High concentrations of Arginine results in the stimulation of N-acetylation of glutamate by acetyl-CoA.

Which enzyme converts the acyl-CoA chain to an acylcarnitine chain, which can be transported across the mitochondrial membrane by carnitine translocase (CAT)?

carnitine palmitoyltransferase 1 (CPT1). If the acyl-CoA chain is long, it needs to be transported across the membrane by the carnitine shuttle. Once inside the mitochondria, CPT2—bound to the inner mitochondrial membrane—converts the acylcarnitine back to acyl-CoA. At this point, acyl-CoA is inside the mitochondria and can now undergo beta oxidation.

What is the common name for a group of glycosphingolipids called monoglycosylceramides which are important components in animal muscle and nerve cell membranes?

cerebrosides; They consist of a ceramide with a single sugar residue at the 1-hydroxyl moiety

Proteins are polymers of amino acids. What name is given to a membrane protein, specifically a transport protein, that has a hydrophilic channel that certain molecules or atomic ions use as a tunnel?

channel protein. Hydrophilic molecules can't penetrate the lipid bilayer and have to go through the protein molecules. Some proteins have an aqueous channel that allows certain types of molecules to pass. Some proteins have a carrier system that allows diffusion of a certain molecule, for example, glucose into the cell.

what is the main role of the Cell membrane?

controls which substances can enter or leave the cell. The primary function of the plasma membrane is to protect the cell from its surroundings. Composed of a phospholipid bilayer with embedded proteins, the plasma membrane is selectively permeable to ions and organic molecules and regulates the movement of substances in and out of cells.

Which of the following inhibits acetyl CoA carboxylase- a rate-limiting enzyme of fatty metabolism? a) Citrate b) ATP c) Malonyl CoA d) Acyl CoA

d) Acyl CoA

The following conditions lead to the hyperammonemia a) Chronic Liver Cirrhosis b) GI bleeding c) Deficiency of CPS I d) All of the above

d) All of the above.

The free ammonium group (NH4+) is derived from the following processes: a) Oxidative deamination of Glutamine catalyzed by enzyme glutamate dehydrogenase b) Deamidation of histidine catalyzed by the enzyme histidase c) Deamidation of serine and threonine by a PLP requiring enzyme serine dehydratase d) All of the above

d) All of the above.

All of the following is complex lipids, except a) Phosphatidic acid b) Cerebroside c) Cardiolipin d) Cholesterol

d) Cholesterol.

Which of the following is not an important precursor of glucose in animals? a) Lactate b) Pyruvate c) Glycerol d) Glucose 6-phosphate

d) Glucose 6-phosphate. The important precursors of glucose in animals are three-carbon compounds such as lactate, pyruvate and glycerol as well as certain amino acids.

Products of glucose oxidation essential for oxidative phosphorylation are a) Pyruvate b) Acetyl co-A c) NADPH and ATP d) NADH and FADH2

d) NADH and FADH2. The end products of Krebs cycle are CO2, NADH, FADH2, ATP and water. The reactants of oxidative phosphorylation are NADH, FADH2, O2, ADP and P.

Lack of appropriate lipid absorption leads to a condition known as a) Metabolic syndrome b) Obesity c) Fatty liver d) Steatorrhea

d) Steatorrhea. Steatorrhea, or fatty stool, occurs when there is too much fat in the stool. Stool or feces contain a mixture of undigested nutrients. These include proteins, fibers, and salts. Stool also typically contains mucous, dead cells, or any other waste the body is able to excrete.

The following require cholesterol EXCEPT a) Bile acid synthesis b) Steroid hormone synthesis c) Membrane fluidity d) Thyroid hormone synthesis

d) Thyroid hormone synthesis. Thyroid hormone synthesis involves the coordinated regulation of signals from the hypothalamus, pituitary, and thyroid. Thyrotropin (TSH) is the main stimulator of thyroid hormone production and its secretion is under the control of thyrotropin-releasing hormone (TRH) secreted by the hypothalamus.

Which enzyme is an extremely efficient metabolic enzyme that catalyzes the interconversion between dihydroxyacetone phosphate (DHAP) and D-glyceraldehyde-3-phosphate (G3P) in glycolysis and gluconeogenesis? a) Hexokinase b) Aldolase c) Galactokinase d) Triosephosphate isomerase

d) Triosephosphate isomerase. The active site of the enzyme houses glutamate and histidine residues that act as general acids and bases, respectively.

The carbamoyl-phosphate is synthesized in the mitochondria and subsequently converted into citrulline. The remainder of the steps occurs in cytosol requiring transport of citrulline into the cytosol. The mitochondrial transporter is a a) symport that co-transport arginine with ornithine b) antiport that exchange arginine with ornithine c) symport that co-transport citrulline with ornithine d) antiport that exchange citrulline with ornithine

d) antiport that exchange citrulline with ornithine

The influence of pH on the rate of enzymatic reactions may be a result of: a. a requirement that the substrate be in a definite ionic form b. a requirement for specific ionic groups on the enzyme protein for enzyme activity c. a requirement for specific groups which do not dissociate at any pH d. 1 and 2 only

d. 1 and 2 only.

In erythrocytes the most abundantly found Phospho ester is- a) Glucose-6 phosphate b) Fructose 1, 6 bisphosphate c) Fructose-6- phosphate d) 2, 3 bisphosphoglycerate

d. 2, 3 bisphosphoglycerate. 2,3-bisphosphoglycerate is mostly found in human red blood cells, or erythrocytes. It has a less oxygen binding affinity to oxygenated hemoglobin than it does to deoxygenated hemoglobin. It also acts to stabilize the oxygen affinity of the hemoglobin in the tense state, since the oxygen affinity is low.

D-Allose, D-Altrose & D-Gulose are examples of? a. 3-carbon D-Aldoses b. 4-carbon D-Aldoses c. 5-carbon D-Aldoses d. 6-carbon D-Aldoses

d. 6-carbon D-Aldoses.

D-Glucose, D-Mannose & D-Galactose are examples of? a. 3-carbon D-Aldoses b. 4-carbon D-Aldoses c. 5-carbon D-Aldoses d. 6-carbon D-Aldoses

d. 6-carbon D-Aldoses.

D-Idose & D-Talose are examples of? a. 3-carbon D-Aldoses b. 4-carbon D-Aldoses c. 5-carbon D-Aldoses d. 6-carbon D-Aldoses

d. 6-carbon D-Aldoses.

D-Fructose is a a. 3-carbon D-Ketose b. 4-carbon D-Ketose c. 5-carbon D-Ketose d. 6-carbon D-Ketose

d. 6-carbon D-Ketose.

D-Psicose, D-Sorbose & D-Tagatose are examples of a. 3-carbon D-Ketoses b. 4-carbon D-Ketoses c. 5-carbon D-Ketoses d. 6-carbon D-Ketoses

d. 6-carbon D-Ketoses.

What's the water content of the teeth? a. 80-85% b. 75-80% c. 70-75% d. 8-10%

d. 8-10%. Human beings are mostly water, ranging from about 75 percent of body mass in infants to about 50-60 percent in adult men and women, to as low as 45 percent in old age. The percent of body water changes with development, because the proportions of the body given over to each organ and to muscles, fat, bone, and other tissues change from infancy to adulthood.

Which of the following is/are essential amino acid(s): a. L-lysine b. L-tryptophan c. L-methionine d. All of these

d. All of these.

In preparation for beta oxidation, which enzyme transfers the acyl group to CoA in the mitochondrial matrix? a. Acyl-CoA synthetase b. Carnitine acyltransferase 1 c. Translocase d. Carnitine acyltransferase 2

d. Carnitine acyltransferase 2. In the mitochondrial matrix, acyl-CoA can be further broken down to acetyl-CoA. This degradation is performed via a metabolic cycle called β-oxidation since the oxidation occurs at a β-carbon atom.

In the electron transport chain, Cytochrome oxidase is a. Complex 1 b. Complex 2 c. Complex 3 d. Complex 4

d. Complex 4. Cytochrome oxidase, also known as complex IV, is the terminal, or final, enzyme of the electron transport system (this does not include ATP synthase). Cytochrome oxidase is a transmembrane molecule found in the mitochondria of eukaryotes and in the cellular space of aerobic prokaryotes.

Which complex in the electron transport chain, uses the electrons and hydrogen ions to reduce molecular oxygen to water? a. Complex 1 b. Complex 2 c. Complex 3 d. Complex 4

d. Complex IV (cytochrome c oxidase). Q passes electrons to complex III (cytochrome bc1 complex), which passes them to cytochrome c (cyt c). Cyt c passes electrons to Complex IV (cytochrome c oxidase), which uses the electrons and hydrogen ions to reduce molecular oxygen to water.

An amino acid which contains a disulfide bond is: a. Lysine b. Methionine c. Homocysteine d. Cystine

d. Cystine. Cystine is made by forming a disulfide bond between two cysteine molecules. It may have anti-inflammatory properties and help protect against toxins and arthritis.

What term is given to the network of protein filaments within some cells that helps the cell maintain its shape and is involved in many forms of cell movement? a. Selectively Permeable b. Heteroplasmy c. Apoptosis d. Cytoskeleton

d. Cytoskeleton. A cytoskeleton is present in the cytoplasm of all cells, including bacteria, and archaea.It is a complex, dynamic network of interlinking protein filaments that extends from the cell nucleus to the cell membrane. The cytoskeletal systems of different organisms are composed of similar proteins. In eukaryotes, the cytoskeletal matrix is a dynamic structure composed of three main proteins,

What term is defined as blockage of a vessel by a clot or foreign material brought to the site by the blood current? a. Prostaglandins b. Right Ventricular Failure c. Cardiomegaly d. Embolism

d. Embolism. An embolism is the lodging of an embolus, a blockage-causing piece of material, inside a blood vessel. The embolus may be a blood clot (thrombus), a fat globule (fat embolism), a bubble of air or other gas (gas embolism), or foreign material.

When glucose enters a cell, in the liver, which enzyme rapidly adds a phosphate to convert it into glucose-6-phosphate? a. Phosphofructokinase-1 b. 3-phosphoglycerate kinase c. Hexokinase d. Glucokinase

d. Glucokinase. A kinase is a type of enzyme that adds a phosphate molecule to a substrate (in this case, glucose, but it can be true of other molecules also). This conversion step requires one ATP and essentially traps the glucose in the cell, preventing it from passing back through the plasma membrane, thus allowing glycolysis to proceed. It also functions to maintain a concentration gradient with higher glucose levels in the blood than in the tissues.

All the following enzymes except one require NADP+ as a coenzyme- a) Glucose- 6-phosphate dehydrogenase b) Cytosolic Isocitrate dehydrogenase c) Malic enzyme d) Glucose-6-phosphatase

d. Glucose-6-phosphatase. Glucose 6-phosphatase is an enzyme that hydrolyzes glucose 6-phosphate, resulting in the creation of a phosphate group and free glucose. Glucose is then exported from the cell via glucose transporter membrane proteins.

The highest phospholipids content is found in ... a) Chylomicrons b) VLDL c) LDL d) HDL

d. HDL. Phospholipids are found in high concentrations in the lining of practically every cell of the body, including brain cells. They help brain cells communicate and influence how well receptors function. Although present in many foods, phospholipids are found in higher concentrations in soy, eggs and the brain tissue of animals.

What is characteristic of the quaternary structure of proteins? a. Amino acid sequence b. Beta-sheets and alpha-helix c. 3-D folding d. Interaction of enzyme with protein

d. Interaction of enzyme with a protein Many proteins are made up of a single polypeptide chain and have only three levels of structure. However, some proteins are made up of multiple polypeptide chains, also known as subunits. When these subunits come together, they give the protein its quaternary structure.

What term is given to bodyweight without body fat? a. Nephrotic Syndrome b. Anaemia c. Pentose Phosphate Pathway d. Lean Body Mass

d. Lean Body Mass. Lean body mass (LBM) is a part of body composition that is defined as the difference between total body weight and body fat weight. This means that it counts the mass of all organs except body fat, including bones, muscles, blood, skin, and everything else.

In the very first step of glycolysis, Hexokinase transfers a phosphoryl group from ATP onto the 6th carbon of glucose to form Glucose-6-phosphate (G6P). What's the necessary cofactor for this reaction with hexokinase? a. sodium b. potassium c. chlorine d. magnesium

d. Magnesium (Mg2+). It is required for the enzyme's function because it shields negative charges on ATPs phosphate oxygen atoms, which makes the terminal phosphate more accessible to nucleophilic attack by the C6-OH group on glucose. Mg2+ is also the cofactor in the conversion of fructose 6-phosphate to fructose 1,6-bisphosphate.

In precursor biosynthesis, which is a major area of carbon metabolism in which the Krebs Cycle is involved, which of the precursors is responsible for the production of Pyruvate? a. Citrate b. Alpha-Ketoglutarate c. Succinyl-CoA d. Malate

d. Malate. Malate dehydrogenases (a.k.a malic enzymes) catabolizes the reaction between Malate and pyruvate. Pyruvate can then be converted to oxaloacetate by the enzyme pyruvate carboxylase.

During alpha-oxidation of fatty acids; a. Energy is generated b. Energy is released c. Energy is absorbed d. No energy produced

d. No energy produced. alpha-Oxidation is a process in which fatty acids are shortened by one carbon atom. The alpha-oxidation sequence of 3-methyl-branched fatty acids starts with an activation to the corresponding CoA-ester. Subsequently this acyl-CoA-ester undergoes a 2-hydroxylation by the peroxisomal phytanoyl-CoA hydroxylase (PAHX).

When considering the effects of pH on the Vmax for an enzyme, it is important to note that: a. It is only the acid-base equilibria of the enzyme which contributes to this effect b. It is only the acid-base equilibria of the substrate which contribute to this effect c. It is only the acid-base equilibria of the catalytic groups of the enzyme which contribute to this effect d. None of the above

d. None of the above. The acid-base equilibria of all of the above groups help to determine the effect pH will have on the enzyme's structure, and hence its activity, and thus the optimum rate at which it can catalyze a reaction (Vmax).

Which of the following amino acids is the precursor for Epinephrine? a. Tryptophan b. Histidine c. Glycine d. Phenylalanine

d. Phenylalanine.

Which enzyme is responsible for the conversion of glucose 6-phosphate to fructose 6-phosphate in the second step of glycolysis? a. Phosphofructokinase-1 b. 3-phosphoglycerate kinase c. Hexokinase d. Phosphoglucose isomerase

d. Phosphoglucose isomerase (PGI) (or phosphoglucomutase). PGI isomerizes the aldose glucose to a ketose-fructose, which is accomplished by opening the ring up using the lysine residue in the active site of the enzyme, followed by ring closure by PGI's active-site histidine.

The toxin in snake venom that causes lysis of RBC is: a. Neurotoxinase b. Haemolyase c. Transketolase d. Phospholipase

d. Phospholipase.

Which of the following amino acids possesses a secondary amino group? a. Hydroxylysine b. Tyrosine c. Valine d. Proline

d. Proline. Proline is the only option here with a secondary amino group and plays a role in cell signaling and molecular recognition (important for cancer cells).

Which of the following amino acids is an "alpha helix terminator"? a. Methionine b. Histidine c. Glycine d. Proline

d. Proline. Proline plays important roles in protein synthesis and structure, metabolism (particularly the synthesis of arginine, polyamines, and glutamate via pyrroline-5-carboxylate), and nutrition, as well as wound healing, antioxidative reactions, and immune responses.

Each of the following lipid types is a membrane constituent EXCEPT: a. Sphingolipids b. Phospholipids c. Gangliosides d. Triglycerides

d. Triglycerides. Triglycerides are storage lipids, not membrane lipids. They are the major form of energy storage in the body and are digested in the small intestines by pancreatic lipase with the aid of bile salts. Sphingolipids are membrane lipids with a sphingosine backbone or a structure similar to it (ceramides, sphingomyelins, glycosphingomyelin etc.)

Which of the following amino acids form Fumarate? a. Alanine, Glycine, Threonine, Cysteine & Serine b. Arginine, Proline, Histidine & Glutamine c. Asparagine & Aspartate d. Tyrosine & Phenylamine

d. Tyrosine & Phenylamine.

Vitamin D-resistant rickets can be caused by: a. failure to absorb photons b. ingestion of a diet of relatively high Ca-P ratio c. failure to ingest vitamin D3 d. a defect in a hydroxylation step

d. a defect in a hydroxylation step. This condition is usually due to a gene mutation resulting in low blood phosphate levels and an inability to hydroxylate Vitamin D1-alpha.

Which of the following molecule is not a gluconeogenic substrate? a) alanine b) oxaloacetate c) glycerol d) acetyl-CoA

d. acetyl-CoA. In humans, the main gluconeogenic precursors are lactate, glycerol (which is a part of the triacylglycerol molecule), alanine and glutamine. Altogether, they account for over 90% of the overall gluconeogenesis. Other glucogenic amino acids as well as all citric acid cycle intermediates, the latter through conversion to oxaloacetate, can also function as substrates for gluconeogenesis.

Which of the following is LEAST likely to affect the Vmax of an enzyme ? a. a change in pH b. a change in temperature c. addition of a non-competitive inhibitor d. addition of a competitive inhibitor

d. addition of a competitive inhibitor. Since competitive inhibitors are reversible, as more substrate is added the inhibitor's effect would eventually be overcome. Thus it is highly unlikely that a competitive inhibitor would affect an enzyme's Vmax.

Concerning the bile acids, which of the following statements is/are CORRECT? a. glycoholic and taurocholic acids are the major conjugated bile acids b. some of the conjugated bile acids are returned to the liver from the intestine via the enterohepatic circulation c. the major lipid components of bile, besides the conjugated bile acids, are phosphatidyl choline and cholesterol d. all of the above

d. all of the above

Bile salts: a. emulsify fats b. are cholesterol derivatives c. adjust the pH of the chime d. all of the above

d. all of the above.

In the liver, glucose is used to: i) create fatty acids ii) cholesterol synthesis iii) glycogen synthesis. a. i only b. i and iii c. iii only d. all of the above

d. all of the above. When carbohydrates are abundant, the liver not only utilizes glucose as the main metabolic fuel but also converts glucose into fatty acids. Hepatocytes also obtain fatty acids from the bloodstream, which are released from adipose tissue or absorbed from food digestion in the GI. Cholesterol and other fats are carried in your bloodstream as spherical particles called lipoproteins.

Products of the intestinal digestion of triglyceride include: a. mainly diglyceride and free acids b. chylomicrons entering the intestinal wall cells provided they are more than 1 micron in diameter c. alpha-glycerol phosphate in the cell wall formed only from triglyceride, glycerol, ATP, and the action of glycerol kinase d. an oil phase consisting of triglyceride and diglyceride and micellar phase of monoglyceride, bile salt and free fatty acid

d. an oil phase consisting of triglyceride, diglyceride and micellar phase of monoglyceride, bile salt and free fatty acid. It's not easy to find information this specific, but little hints from Lippincott's and other online resources indicate that mixed micelles are formed during the breakdown of triglycerides into monoglycerides. Pancreatic lipase is a water soluble enzyme, and bile salts are needed to break up the triglyceride oil droplets into micelles-- to make them easier to digest. We're not too sure if diglycerides are present, but this is the most likely answer.

In the urea cycle, which enzyme catalyzes the combination of citrulline with aspartate? a. glutamate dehydrogenase b. carbamoyl phosphate synthetase c. ornithine transcarbamylase d. argininosuccinate synthetase

d. argininosuccinate synthetase 1 (ASS1). Citrulline is transported out of the mitochondria into the cytoplasm and combines with aspartate to form argininosuccinate. This reaction requires ATP.

The side chain of which amino acid residue in the active site of trypsin forms an ionic bond to basic amino acids substances? a. hydroxylysine b. proline c. lysine d. aspartate

d. aspartate.

A carbon atom in a molecule becomes an asymmetric center when it bears: a. a pair of like and a pair of unlike substituents b. a double bond c. five different substituents d. four different substituents

d. four different substituents. A carbon with four different groups (substituents) cannot have any plane of symmetry. Thus it is asymmetric or chiral.

Hydrolysis of sucrose yields: a. glucose only b. galactose and glucose c. maltose and glucose d. fructose and glucose

d. fructose and glucose. These two monomers help to make up sucrose, so hydrolysis of sucrose would produce both fructose and glucose.

Starch and Glycogen both are polymers of: a. fructose b. glucose-1-phosphate c. mannose d. glucose

d. glucose. Glucose forms many polymers by alpha-1,4-linkages and beta-1,4-linkages. Those formed by alpha-1,4-linkages include amylose, amylopectin, which combine to form starch (in plants) ; animals have glycogen. Cellulose is a special one in cell walls that has beta-1,4-linkages.

Which enzyme catalyzes the final reaction of beta-oxidation? a. acyl-CoA dehydrogenase b. enoyl-CoA hydratase c. hydroxyacyl-CoA dehydrogenase d. ketothiolase

d. ketothiolase. This happens as the thiol group of another CoA molecule is transferred to the ketoacyl-CoA. Thus, acetyl-CoA is split off and an acyl-CoA that is now shorter by 2 carbon atoms remains. The latter can enter another oxidation cycle.

There are two distinct stereoisomeric modifications of glucose which are interconverted in aqueous solution to yield an equilibrium mixture. This phenomenon is termed: a. polarisation b. amphoterism c. optical isomerism d. mutarotation

d. mutarotation. Mutarotation deals with the interconversion or mixing of cyclic epimers (called anomers) in a solution and how these change how much the solution will rotate plane-polarized-light (optical rotation). In the case of glucose, if you take either the pure alpha-D-Glucose or the pure beta-D-glucose anomer, and place it in a solution, either one will convert to the other until an equilibrium concentration of about 64% Beta, 36% alpha is reached. This is mutarotation.

A prosthetic group of a protein is a non-protein structure that is: a) a ligand of the protein. b) a part of the secondary structure of the protein. c) a substrate of the protein. d) permanently associated with the protein.

d. permanently associated with the protein. Many proteins contain only amino acids and no other chemical groups, and they are called simple proteins. The non-amino part of a conjugated protein is usually called its prosthetic group.

The tissue of starved animals would contain which lipid? a. glycolipids b. triglycerides c. cholesterol d. phospholipids

d. phospholipids.

Glucose and Mannose are epimers. This means that: a. one is an aldose, the other a ketose b. one is a pyranose, the other a furanose c. they rotate plane-polarized light in opposite directions d. they differ only in the configuration of one carbon atom

d. they differ only in the configuration of one carbon atom. Mannose and Galactose are epimers of Glucose. Glucose differs from Mannose at carbon #2, and from galactose at carbon #4. Remember epimers are molecules which are almost identical except for the configuration of atoms around one carbon; e.g D-galactose and D-glucose which differ at carbon #4.For the record, both glucose and mannose are aldoses (aldehyde sugars).

In the citrate shuttle system of fatty acid biosynthesis, Acetyl-CoA reacts with oxaloacetate to give citrate. What enzyme transports citrate from mitochondria to cytosol? a. Fatty acid synthetase b. Pyruvate dehydrogenase c. Acetyl CoA carboxylase d. tricarboxylate translocase

d. tricarboxylate translocase

Myoglobin and the subunits of hemoglobin have: a) no obvious structural relationship. b) very different primary and tertiary structures. c) very similar primary and tertiary structures. d) very similar tertiary structures, but different primary structures.

d. very similar tertiary structures, but different primary structures. Hemoglobin is a tetramer composed of two each of two types of closely related subunits, alpha and beta. Myoglobin is a monomer (so it doesn't have a quaternary structure at all). Myoglobin binds oxygen more tightly than does hemoglobin.

What happens when an IPSP is generated after EPSP? a. A threshold event takes place. b. Effect of subthreshold is enhanced. c. Action potential is reached. d. Membrane is more depolarized. e. Membrane is hyperpolarized.

e. Membrane is hyperpolarized. Hyperpolarization is a change in a cell's membrane potential that makes it more negative. It is the opposite of a depolarization. It inhibits action potentials by increasing the stimulus required to move the membrane potential to the action potential threshold.

All the below would be possible occurrences in case of signal build up EXCEPT? a. Membrane is in depolarized state. b. Action potential will reach if number of EPSP>IPSP. c. They can reach action potential as a result of EPSP. d. IPSP can hyperpolarize the membrane. e. They can reach action potential as a result of IPSP.

e. They can reach action potential as a result of IPSP.

What's the main function of triglycerol in our bodies?

energy storage; Triglycerides are a type of fat that plays a major role as an energy source when they are metabolized in the human body. They are very rich in energy, containing double the energy of either carbohydrates or proteins that can also be used to supply energy to the body.

What are the primary fuels for resting muscle?

free fatty acids from adipose tissue and ketone bodies from the liver. Skeletal muscles use free fatty acids, ketone bodies, or glucose as fuel, depending on the degree of muscular activity. Moderately active muscles use blood glucose, fatty acids and ketone bodies. Maximally active muscles, the demand for ATP is great and blood flow cannot provide O2 and fuels fast enough to produce the necessary ATP by aerobic respiration alone.

Together, the pancreatic lipases and bile salts break down triglycerides into?

free fatty acids. These fatty acids can be transported across the intestinal membrane. However, once they cross the membrane, they are recombined to again form triglyceride molecules. Within the intestinal cells, these triglycerides are packaged along with cholesterol molecules in phospholipid vesicles called chylomicrons.

What term is given to the process of converting pyruvic acid to glucose-6-phosphate?

gluconeogenesis. Acetyl CoA supplies carbons for fatty acid synthesis in the cytosol.

ATP for muscle contraction is produced by metabolism of?

glucose and fatty acids. During short bursts of activity the ATP demand exceeds the rate of production, then anaerobic glycolysis produces ATP, lactate and H+. Muscle uses ATP for mechanical work. Metabolism in skeletal muscle is specialized to generate ATP as the immediate source of energy.

Monosaccharides are the building blocks of disaccharides (such as sucrose and lactose) and polysaccharides (such as cellulose and starch). What are some examples of Monosaccharides?

glucose, ribose, mannose, fructose, galactose; the simplest carbohydrates

Ammonia has a high pK value and occurs in the blood mainly as ammonium ions (NH+4). Free ammonia can easily enter the brain cells and the mitochondria leading to the formation of?

glutamate from NH+4 and alpha-ketoglutarate. This results in the depletion of alpha-ketoglutarate from the TCA cycle. This lowers the rate of glucose oxidation and therefore the brain becomes deprived of glucose, which is its major fuel source.

The first step in lipid metabolism is the hydrolysis of the lipid in the cytoplasm to produce?

glycerol and fatty acids. Since glycerol is a three carbon alcohol, it is metabolized quite readily into an intermediate in glycolysis, dihydroxyacetone phosphate. The last reaction is readily reversible if glycerol is needed for the synthesis of a lipid.

Glycerol serves as a substrate for gluconeogenesis in the liver. It's acted upon by ___________ _____________ enzyme, consuming one ATP in the process, to produce L-glycerol-3-phosphate (L-G3P).

glycerol kinase. L-G3P will be converted to a dihydroxyacetone phosphate using glycerol phosphate dehydrogenase. An NAD+ is also converted to NADH in the process.

Glucose-1-phosphate is cleaved from glycogen by the enzyme _______________ ______________ which then can be converted to glucose-6-phosphate.

glycogen phosphorylase. The process of liberating glucose from glycogen is known as glycogenolysis. This process is essentially the opposite of glycogenesis with two exceptions: (1) there is no UDP-glucose step, and (2) a different enzyme, glycogen phosphorylase, is involved.

If a person is in an anabolic state, such as after consuming a meal, most glucose-6-phosphate within the myocytes (muscle cells) or hepatocytes (liver cells) is going to be stored as?

glycogen. Glycogen is mainly stored in the liver and the muscle. It makes up ~6% of the wet weight of the liver and only 1% of muscle wet weight. However, since we have far more muscle mass in our body, there is 3-4 times more glycogen stored in muscle than in the liver.

What are the 2 main forces involved in the formation of edema?

hydrostatic pressure and oncotic pressure; Hydrostatic pressure is the pressure generated by fluids while oncotic pressure is the osmotic pressure generated by proteins in blood plasma.

Also employed to combat hypoglycemia is the ___________, which has glucoreceptors to trigger the autonomic nervous system to secrete epinephrine, and the anterior pituitary to release adrenocorticotropic hormone (ACTH) and growth hormone (GH).

hypothalamus. ACTH increases cortisol synthesis and releases in the adrenal cortex. Catecholamines such as epinephrine and norepinephrine playing a supporting role in the fasted state as well, by causing similar cellular responses as those produced by glucagon.

Where is myoglobin mainly found?

in muscles. Myoglobin and hemoglobin are heme proteins whose physiological importance is principally related to their ability to bind molecular oxygen. Myoglobin is a monomeric heme protein found mainly in muscle tissue where it serves as an intracellular storage site for oxygen. During periods of oxygen deprivation oxymyoglobin releases its bound oxygen which is then used for metabolic purposes.

What is absolutely necessary for the formation of massive generalized oedema a) renal retention b) blocked lymphatics c) increased plasma hydrostatic pressure d) increased plasma colloid osmotic pressure

increased plasma hydrostatic pressure. First, in relation to edema associated with MCD or other non-inflammatory conditions resulting in massive proteinuria, an increase in transcapillary oncotic pressure gradient is the single most important driver of edema formation.

Lipid metabolism begins in the ________________ where ingested triglycerides are broken down into smaller chain fatty acids and subsequently into monoglyceride molecules by pancreatic lipases, enzymes that break down fats after they are emulsified by bile salts.

intestine. When food reaches the small intestine in the form of chyme, a digestive hormone called cholecystokinin (CCK) is released by intestinal cells in the intestinal mucosa. CCK stimulates the release of pancreatic lipase from the pancreas and stimulates the contraction of the gallbladder to release stored bile salts into the intestine. CCK also travels to the brain, where it can act as a hunger suppressant.

What are the types of Apoptosis Pathways?

intrinsic and extrinsic. The two major types of apoptosis pathways are "intrinsic pathways," where a cell receives a signal to destroy itself from one of its own genes or proteins due to detection of DNA damage; and "extrinsic pathways," where a cell receives a signal to start apoptosis from other cells in the organism.

If excessive acetyl CoA is created from the oxidation of fatty acids and the Krebs cycle is overloaded and cannot handle it, the acetyl CoA is diverted to create?

ketone bodies. These ketone bodies can serve as a fuel source if glucose levels are too low in the body. Ketones serve as fuel in times of prolonged starvation or when patients suffer from uncontrolled diabetes and cannot utilize most of the circulating glucose. In both cases, fat stores are liberated to generate energy through the Krebs cycle and will generate ketone bodies when too much acetyl CoA accumulates.

In a subsequent reaction, malate is converted to pyruvate, NADPH is produced, and CO2 is released. What enzyme catalyses this reaction?

malic enzyme (or NADP+ - dependent malate dehydrogenase). The pyruvate produced reenters the mitochondrion and is reutilized. The NADPH supplies reducing equivalents for reactions on the fatty acid synthase complex. It's produced by malic enzyme and the pentose phosphate pathway.

What term is given to a metabolite that, when bound to the allosteric site of an enzyme, alter its kinetic characteristics?

modulator. The modulators of allosteric enzymes may be either stimulatory or inhibitory. Many enzymes do not demonstrate hyperbolic saturation kinetics or typical Michaelis-Menten kinetics.

In the body, water moves through semi-permeable membranes of cells and from one compartment of the body to another by a process called osmosis. What term is given to the pressure that would have to be applied to a pure solvent to prevent it from passing into a given solution by osmosis?

osmotic pressure; It is often used to express the concentration of the solution. Osmosis is basically the diffusion of water from regions of higher concentration to regions of lower concentration, along an osmotic gradient across a semi-permeable membrane. As a result, water will move into and out of cells and tissues, depending on the relative concentrations of the water and solutes found there.

Glucose primarily becomes available in the blood as a result of glycogen breakdown, or from its synthesis from noncarbohydrate precursors (gluconeogenesis). In gluconeogenesis, each of the carbon precursors must first be converted into ________________ which is the 4 carbon precursor for the process.

oxaloacetate. Gluconeogenesis in mammals occurs during fasting, starvation or intense exercise, and is an endergonic process, which requires ATP and GTP. The liver's capacity to store glucose as glycogen is only sufficient to supply the brain for about half a day when fasting. Isotopic labelling studies have shown that gluconeogenesis is responsible for 28% of total glucose production after 12 hrs of fasting, and for almost all glucose by 66 hrs.

Energy metabolism, precusor biosynthesis & Gluconeogenesis are the 3 major areas of carbon metabolism in which the Krebs cycle is involved. Acetyl-CoA enters the Krebs cycle by combining with what?

oxaloacetic acid. The combination forms the six-carbon acid called citric acid. Citric acid undergoes a series of enzyme-catalyzed conversions. The conversions, which involve up to ten chemical reactions, are all brought about by enzymes.

In proteins, amino acids are joined covalently by _____________ ____________, which are amide linkages between the alpha -carboxyl group of one amino acid, and the alpha -amino group of another.

peptide bonds. For example, valine and alanine can form the dipeptide valyl alanine through the formation of a peptide bond. Peptide bonds are not broken by conditions that denature proteins, such as heating or high concentrations of urea. Prolonged exposure to a strong acid or base at elevated temperatures is required to hydrolyse these bonds non-enzymatically.

In cases where fatty acid chains are too long to enter the mitochondria, where can beta oxidation take place?

peroxisomes; First, fatty acid protein transporters allow fatty acids to cross the cell membrane and enter the cytosol, since the negatively charged fatty acid chains cannot cross it otherwise. Then, the enzyme fatty acyl-CoA synthase (or FACS) adds a CoA group to the fatty acid chain, converting it to acyl-CoA.

The principal components of the plasma membrane are lipids (phospholipids and cholesterol), proteins, and carbohydrates. What are the 3 types of endocytosis?

phagocytosis, pinocytosis & receptor-mediated endocytosis. In phagocytosis or " cellular eating," the cell's plasma membrane surrounds a macromolecule or even an entire cell from the extracellular environment and buds off to form a food vacuole or phagosome. The plasma membrane protects intracellular components from the extracellular environment.

All aminotransferases require the prosthetic group ___ as a cofactor, which is derived from pyridoxine (vitamin B6).

pyridoxal phosphate (PLP). Pyridoxal phosphate enzymes labilize one of three bonds at the α-carbon atom of an amino acid substrate.

All aminotransferases require the prosthetic group pyridoxal phosphate (PLP) as a cofactor, which is derived from?

pyridoxine (vitamin B6). Pyridoxal phosphate includes a pyridine ring that is slightly basic as well as a phenolic hydroxyl group that is slightly acidic. PLP acts as a coenzyme in all transamination reactions, and in certain decarboxylation, deamination, and racemization reactions of amino acids. In these reactions, the PLP reacts with glutamate, which transfers its alpha-amino group to PLP to make pyridoxamine phosphate (PMP).

What enzyme catalyzes the the conversion of pyruvate to oxaloacetate?

pyruvate carboxylase. Pyruvate carboxylase (PC) is a biotin-containing enzyme that catalyzes the formation of oxaloacetate in the presence of an allosteric activator, acetyl CoA, from pyruvate.

What enzyme converts pyruvate to acetyl Coenzyme A (CoA)?

pyruvate dehydrogenase. Glucose enters liver cells and is converted via glycolysis to pyruvate,which enters mitochondria. Pyruvate is converted to acetyl coenzyme A (CoA) by pyruvate dehydrogenase and to oxaloacetate (OAA) by pyruvate carboxylase. Because acetyl CoA cannot directly cross the mitochondrial membrane and enter the cytosol to be used for the process of fatty acid synthesis, acetyl CoA and Oxaloacetate condense to form citrate, which can cross the mitochondrial membrane.

Amino acids are the building blocks of proteins. Glucogenic amino acids are degraded to ___ that will become glucose.

pyruvate or other Krebs Cycle intermediates. In energy metabolism, high energy electrons are stripped from small metabolite molecules containing carbon, oxygen and hydrogen and passed to the electron transport chain.

What's the most common cause of secondary hypertension? a. Renal disease b. Right Ventricular Failure c. Cardiomegaly d. Embolism

renal disease. Secondary hypertension (secondary high blood pressure) is high blood pressure that's caused by another medical condition. Secondary hypertension can be caused by conditions that affect your kidneys, arteries, heart or endocrine system.

What's the term for the ratio of a mineral's weight compared with the weight of an equal volume of water?

specific gravity. Ideally, urine specific gravity results will fall between 1.002 and 1.030 if your kidneys are functioning normally. Specific gravity results above 1.010 can indicate mild dehydration. High urine specific gravity can indicate that you have extra substances in your urine, such as: glucose.

Fats (or triglycerides) within the body are ingested as food or synthesized by adipocytes or hepatocytes from carbohydrate precursors. What is the complex alcohol backbone for membrane lipids?

sphingosine. Lipid metabolism entails the oxidation of fatty acids to either generate energy or synthesize new lipids from smaller constituent molecules. Lipid metabolism is associated with carbohydrate metabolism, as products of glucose (such as acetyl CoA) can be converted into lipids.

Why is ATP termed as a high energy compound?

the presence two anhydride bonds in the triphosphate unit. Hydrolysis of terminal phosphates yields a large negative change in free energy ATP + H2O → ADP + Phosphate gives -7.3 kcal/mol.

Insulin is a polypeptide hormone produced by β-cells in the pancreatic islets of Langerhans. What is the result of the interplay of the hormones insulin and glucagon?

the regulation of blood glucose; Blood glucose levels remain tightly regulated within the human body because glucose is the primary nutrient that supplies tissues, such as the brain, retina, muscles, in quantities for optimal function. However, if the blood glucose level exceeds what is needed for functioning the excess must be mobilized out of the bloodstream and into storage

What is the rate of creation of product over time, measured as the concentration of product per time?

the velocity of a reaction. The time is a critical consideration when measuring velocity. In a closed system (in which an enzyme operates), all reactions will advance towards equilibrium. Enzymatically catalyzed reactions are no different in the end result from non-enzymatic reactions, except that they get to equilibrium faster.

The synthesis of glycogen from glucose is a process known as glycogenesis. Glucose-6-phosphate is not inserted directly into glycogen in this process. There are a couple of steps before it is incorporated. First, glucose-6-phosphate is converted to glucose-1-phosphate and then converted to?

uridine diphosphate (UDP)-glucose. UDP-glucose is inserted into glycogen by either the enzyme, glycogen synthase (alpha-1,4 bonds), or the branching enzyme (alpha-1,6 bonds) at the branch points.

How is body mass index (BMI) calculated?

{weight in kg} divided by {height in meters square}

In this ketone synthesis reaction, excess acetyl CoA is converted into hydroxymethylglutaryl CoA (HMG CoA). HMG CoA is a precursor of cholesterol and is an intermediate that is subsequently converted into ____________________, the primary ketone body in the blood.

β-hydroxybutyrate. Excess acetyl CoA is diverted from the Krebs cycle to the ketogenesis pathway. This reaction occurs in the mitochondria of liver cells. The result is the production of β-hydroxybutyrate, the primary ketone body found in the blood.

What is the amount of free energy liberated by ATP?

∆G= -7.3 kcal/mol or -7,300 cal/mol. Major activities promoted by ATP: - locomotion, - membrane transport, - signal transduction & - nucleotide synthesis


Related study sets

SMART Goal And Crucial Conversations

View Set

Unit 6 Applied Discussion Box Notes 35-44

View Set

Chapter 38: Agents to Control Blood Glucose Levels

View Set

abeka 7th grade history section review 2.4

View Set

2020/2021 ASVAB FOR DUMMIES (PAPER VERSION 1)

View Set

Chapter 1: Managerial Accounting and Cost Concepts

View Set